[Ответить в тред] Ответить в тред

03/04/16 - Набор в модераторы 03.04 по 8.04
26/03/16 - Конкурс: Помоги гомункулу обрести семью!
15/10/15 - Набор в модераторы 15.10 по 17.10



[Назад][Обновить тред][Вниз][Каталог] [ Автообновление ] 797 | 41 | 207
Назад Вниз Каталог Обновить

ШАД Аноним # OP  08/03/16 Втр 00:12:35  386272  
14573851557660.jpg (8Кб, 250x250)
Для всех желающих поступить в Школу анализа данных при Яндексе. Обсуждаем вступительные задачи прошлых лет, делимся материалами.
Аноним # OP  08/03/16 Втр 00:21:24  386275
14573856849330.jpg (350Кб, 1224x1632)
14573856849401.jpg (176Кб, 960x1280)
14573856849442.jpg (722Кб, 2560x1920)
Варианты прошлого года
Аноним 08/03/16 Втр 00:23:45  386278
Как я понимаю, для студента технической специальности это по отдельности не особо проблема решить, проблема в том, что надо решить все и правильно?
Аноним 08/03/16 Втр 00:26:33  386279
http://www-bcf.usc.edu/~lototsky/PiMuEp/Putnam1985-2000.pdf

Здесь много олимпиадных задач с решениями. Некоторые задачи для экзамена в ШАД брались отсюда.
Аноним 08/03/16 Втр 00:29:11  386281
>>386278
Там надо набрать определенное количество баллов. Задач обычно 8, каждая задача оценивается в один балл. Проходной балл что-то около 3,5 или 4. То есть, например, можно решить 3 задачи целиком и еще две по половинке, как раз 4 и будет. Правда, чем больше задач ты решил, тем легче будет на собеседовании потом.
Аноним 08/03/16 Втр 03:27:26  386288
Бамп
Аноним 08/03/16 Втр 09:58:07  386298
>>386275
Задания для даунов... ))
На экзаменах в универе задачи заковыристее давали.

глядя на задачи, в уме складывается программная их реализация...

Аноним 08/03/16 Втр 10:54:18  386301
14574236584040.jpg (234Кб, 628x1298)
>1.Сформируйте систему линейных уравнений (то есть задайте матрицу коэффициентов A и свободный вектор b) для многочлена первой степени, который должен совпадать с функцией f в точках 1 и 15. Решите данную систему с помощью функции scipy.linalg.solve. Нарисуйте функцию f и полученный многочлен. Хорошо ли он приближает исходную функцию?
Я решил, получилось 3.43914511, -0.18692825

Второе часть не могу понять

>2.Повторите те же шаги для многочлена второй степени, который совпадает с функцией f в точках 1, 8 и 15. Улучшилось ли качество аппроксимации?
Где взять многочлен второй степени?
Аноним 08/03/16 Втр 12:59:19  386311
>>386301
Берешь два вместо n
Аноним 08/03/16 Втр 12:59:42  386312
>>386279
Хуясе, на кого они рассчитаны, на математиков?
Аноним 08/03/16 Втр 13:14:48  386314
>>386298
Там задач на программирование максимум две
Аноним 08/03/16 Втр 13:16:11  386315
>>386312
На студентов математиков американских вузов
Аноним 08/03/16 Втр 13:21:03  386316
>>386314
Программисты без матана -- говно собачье, а не программисты
Аноним 08/03/16 Втр 13:35:50  386318
>>386311
>Берешь два вместо n
Как будет выглядит уравнение?

Уравнение для первого пункта

a 1 + b = f(1) и a 15 + b = f(15)


Аноним 08/03/16 Втр 13:36:19  386319
>>386318
>a 1 + b = f(1) и a 15 + b = f(15)
фикс
Аноним 08/03/16 Втр 13:37:02  386320
>>386319
Макаба не хочет отображать знак умножения между а 1 и а 15
Аноним 08/03/16 Втр 14:31:59  386326
Для решения половины заданий хватит знаний 1-го курса мехмата
Аноним 08/03/16 Втр 15:32:58  386332
>>386316
А кто спорит
Аноним 08/03/16 Втр 15:34:31  386333
>>386326
Так и есть. Никто и не говорит, что задачи суперсложные.
Аноним 08/03/16 Втр 15:52:06  386336
В прошлом году, помнится, активность шад-треда была выше. Неужели никто из здесь сидящих не хочет поступить?
Аноним 08/03/16 Втр 16:20:16  386337
14574432170360.jpg (487Кб, 1224x1632)
Аноним 08/03/16 Втр 17:24:55  386349
>>386275
кто-нибудь может подсказать, как решать задачу 5 про массивы из варианта от 24 мая?
Аноним 08/03/16 Втр 18:25:42  386358
Вроде знакомые слова, но чувствую, что нихуя не могу решить. Всё очень плохо?
1 курс пми
Аноним 08/03/16 Втр 18:50:50  386362
Хм, такое ощущение, что для сдачи экзамена на проходной балл даже необязательно учиться в техвузе, хватит недели подготовки и базовой программы по матану и линейке какого-нибудь экономического вуза.
Аноним 08/03/16 Втр 19:14:57  386368
>>386349
Все, я сам решил
Аноним 08/03/16 Втр 19:15:38  386370
>>386358
Учитывая, что в шад принимают только 2 курс и старше - нет
Аноним 10/03/16 Чтв 20:33:05  386821
>>386370
То есть если я мегаохуенный гений-самоучка первокурсник,то меня не возьмут?
Аноним 10/03/16 Чтв 20:44:16  386827
поясните, что лучше выбрать - шад или технопарк мэйла в бауманке и почему?
Аноним 10/03/16 Чтв 20:53:22  386829
>>386272 (OP)
Зашёл сюда, чтобы создать этот тред. ОП молодец, старый тред хорошо помог в поиске литературы. Сейчас попробую порешать эти варианты.
Сам пробовал в том году, но соснул. Пытался ботать, чтобы сделать камбэк. Давайте делиться годнотой. Для начала вам классика: Садовничий с его олимпиадными задачами, и Кнут с его Конкретной математикой.
Ну а вообще скажу так: подтянуть базу вы сможете, но решать такие задачи, значит иметь смекалку определённую, так как задачи околоолимпиадные. Поэтому то, что там сверху пишут про программу первого курса любого дерьмовуза и т.д. -- херня. Большинство задач отсюда не имеет типовых решений.
Аноним 10/03/16 Чтв 21:48:21  386834
>>386827
Если ты не учишься в бауманке, то технопарк.
И еще в него во много раз легче поступить.
Но ШАД НАМНОГО круче в плане перспектив и непосредственно твоей зп будущей.
Аноним 10/03/16 Чтв 21:48:45  386835
>>386834
если учишься в бауманке.
фиксанул.
Аноним # OP  10/03/16 Чтв 22:44:27  386841
>>386829
Я сам соснул в прошлом году, решал вариант за 7 июня, решил полностью только 2 и 3 задачи. А мои однокурсники решали 24 мая, на мой взгляд их вариант был гораздо легче. Обидна.
ОП
Аноним # OP  10/03/16 Чтв 22:45:26  386842
>>386821
Нет, увы. Да и зачем тебе, на первых курсах времени и так не хватает.
Аноним # OP  10/03/16 Чтв 22:48:48  386843
https://habrahabr.ru/company/yandex/blog/262543/
Решение варианта за 24.05.15
Аноним 12/03/16 Суб 17:24:09  387049
бампецкий
Аноним 13/03/16 Вск 01:04:12  387097
Если здесь есть аноны, закончишие ШАД, расскажите, что это дало вам, какие перпективы вообще итд
Аноним 13/03/16 Вск 23:07:33  387203
Сколько вообще бюджетных мест?
Аноним 17/03/16 Чтв 13:53:04  387614
Бамп
Аноним 20/03/16 Вск 02:32:02  388031
Братишки, а покидайте годных вариантов письменного шада, пожалуйста.
Аноним # OP  20/03/16 Вск 03:01:30  388032
>>388031
https://yandexdataschool.ru/admission
Раздел Подготовка
Аноним 20/03/16 Вск 03:06:55  388033
>>388032
Эти варианты уже давно прорешаны, хотелось бы ещё порешать. Кроме тех, что уже кидали в тред.
Аноним 20/03/16 Вск 21:56:28  388191
>>386301
как ты получил 3.43914511, -0.18692825? распиши целиком матрицу
Аноним 20/03/16 Вск 22:25:45  388193
бамп
Аноним 21/03/16 Пнд 22:21:27  388333
>>388193
бамп
Аноним 23/03/16 Срд 16:00:04  388639
Бамп. Накидайте книг, а то не делитесь.
Аноним 24/03/16 Чтв 17:03:59  388809
>Поступающие в филиалы и на заочное отделение сдадут экзамен онлайн 5 июня. Время начала экзамена станет известно позже.
Чё за херня, анон?
Аноним 24/03/16 Чтв 19:08:00  388847
Про старателей клёвая задачка.
Аноним 24/03/16 Чтв 19:30:06  388853
>>388847
Ну-ка, подскажи, что делать. С двумя старателями очевидно. Для трёх старателей можно рассмотреть случай, когда у двух одинаковые кучки, и тоже норм. А дальше как?
Аноним 24/03/16 Чтв 23:57:16  388887
14588530365640.png (221Кб, 1264x1138)
>>388853
Аноним # OP  25/03/16 Птн 12:16:28  388925
>>388639
https://vk.com/doc353651729_437336981
Вот например. В ВК в док-ах полно книг. Кострикин тот де самый.
Аноним # OP  25/03/16 Птн 16:33:45  388981
>>388967
Пшел нахуй пидрила.
Алсо, вот еще хороший НМУшный учебник по алгебре. Я там находил даже какую-то задачу из вступ. экзов в ШАД(правда без решения)
https://vk.com/doc8594309_424355510
Аноним 25/03/16 Птн 17:58:31  388998
>>386275
Посоны, а поясните как решить задачу про детерминанты
det(E-AB) = det(E-BA)
Аноним 25/03/16 Птн 19:45:14  389014
>>388887
Спасибо, анон. Вот как так думать? Я сколько пытаюсь, вроде не туплю, основы знаю, но иногда хер подгадаешь. Эту задачу пытался решить как возвратную (Ханойская башня, и т.д.). Рекурсии, сведение к задаче n - 1, мат.индукция, это вот всё.
>>388925
Садовничий есть. Кострикин стоит того? Он большой. Я вот только закончил читать Куроша. Он более общий, мне кажется, что КПД от такого подхода больше, а Кострикин скорее справочная литература. Хотя хз.
Путмановские олимпиады советовали уже? Ищите их, он мощные (надо уметь в английский). Ещё дляя освежения памяти для лёгкого чтения хватайте "Краткий курс высшей математики" Демидович, Кудрявцев.
И не жадичайте годнотой, тут всего парра анонов, готовая въёбывать. Вряд ли они вам помешают. Где лучше мне почитать про хитрые методы взятия интегралов? У Фихтенгольцца?
>>388809
>пающие в филиалы и на заочное отделение сдадут экзамен онлайн 5 июня. Время начала экзамена станет известно позже.
>Чё за херня, анон?
И вот это вот продублирую. Зайдите на сайт. Что думаете? Лично мне было бы лучше в аудитории, так как там мозга лучше работает. С онлайн же у людей будет вольфрам и иже с ним, что мне кажется не очень хорошо. Кто хорошо гуглит, тому бонус.
Аноним 25/03/16 Птн 19:46:51  389015
>>388998
Единичная матрица -- любая матрица, умноженная на обратную. Попробуй представить единичную в виде АА^-1, попреобразовыйвай. Не выйдет -- пиши, но там несложно. Лучше самому проделать.
Аноним 25/03/16 Птн 19:55:24  389017
14589249245620.png (32Кб, 252x232)
>>389015
Но ведь матрица А может быть необратима.
Аноним 25/03/16 Птн 19:58:06  389018
>>389017
Ты прав, анон. Лучше тут глянуть в условие задачи. Как доказать для необратимых я не знаю.
Аноним 25/03/16 Птн 19:59:33  389019
>>389018
Проглядел я задание. Да, там не указано. Хорошо, для обратимых доказали. Кто тут умный? Что дальше делать? Алсо засчитали бы только за случай с обратимыми какие-нибудь баллы?
Аноним 25/03/16 Птн 20:01:52  389021
>>389014
>Кострикин стоит того?
У Кострикина нужен по идее только 2 том и часть первого. Он простой и понятный, кмк, подходит для новичков хорошо. Есть еще Городенцев(>>388981), он продвинутей.
Как тебе Курош кстати? У меня почему-то возникло впечатление что он сложноват немного, но я недолго читал его.

>Путмановские олимпиады советовали уже?
4 постом скидывал.

>И вот это вот продублирую. Зайдите на сайт. Что думаете?
Думаю писать в аудитории, чтоб поступить на очку, потом если что, на заочку перевестись можно. А онлайн экзамен это как-то странно, люди будут инетом пользоваться, так что по идее вариант должен быть сложнее.

ОП
Аноним 25/03/16 Птн 20:29:14  389024
>>389021
У Куроша несколько книг. Я читал "курс высшей алгебры". Три года назад её проходил, сейчас что-то вспоминал, что-то новое узнавал. Мне он очень вкатил. Всё было более-менее понятно (доказательства я правда иногда скипал, просто ловил идею, а в подробности не лез). Но вот в последних главах он начал рассказывать про поля, кольца и группы. На пальцах это я понял, зачем надо тоже понял, но всякие вывода остались для меня магией. Решил не тратить много сил на разбирательство, так как в экзамене этого всё равно нет, и пролистал. Может когда-нибудь углублюсь в эти дебри. Сам я физик, математика у меня более общая была.
Кстати, вот тут и я тоже >>386829 уже успел посоветовать Садовничего, забыл уже. Как бы в советах не пойти на третий круг. Про графы бывают часто задачи, неплохо быы их подучить. Могу посоветовать на степике (stepic) курс по графам просмотреть. Основы в начале, алгоритмы стандартные.

Задачи на алгоритмы --
Аноним 25/03/16 Птн 20:29:28  389025
14589269690260.png (32Кб, 732x152)
>>389019
> Кто тут умный?
Я умный.

Кстати, это доказывает справедливость выражения для любых матриц A(n×m) и B(m×n), а не только квадратных.
Аноним # OP  26/03/16 Суб 01:23:36  389053
>>389025
А ты походу реал умный. Как вообще можно было додуматься до такого представления матриц??
Аноним 26/03/16 Суб 08:56:56  389073
>>389025
>>389019
>>389018
>необратимая
Пилю лайфхак с нормальной математикой. Обратимые матрицы (и даже диагонализуемые) всюду плотны среди всех матриц (открыты по зарисскому), так как это условие Det не равен 0.
Поэтому если мы хотим проверить какое-то равенство с непрерывными функциями от матриц, то достаточно проверять для обратимых.
Это вообще один из главных трюков в линале, который на удивление редко упоминают. Почти любое равенство так доказывается, ибо для диагонализуемых обычно все очевидно. Есть мнение, что любую задачу в линале можно решить, используя это и структурную теорему о модулях над кольцом главных идеалов (обычно Жордановой нормальной формы хватает).
Аноним 26/03/16 Суб 14:43:10  389121
>>389073
Спасибо, анон. А теперь скажи, где таким обмазаться. Кострикин? Винберг?
Аноним 26/03/16 Суб 21:41:32  389165
>>386834
а если я считаю, что лучше намного больше усердствовать, но попробовать и поступить в шад?
а вообще, это все планы-мечты, тк я пока первокур. И, кстати, вопрос тогда уж - в шад лучше идти курсе на 3-4? И не мог бы ты пояснить за направления в нем? спасибо ^^
Аноним 26/03/16 Суб 21:59:46  389169
>>389121
ЖНФ и модули в Винберге, да. Самый приятный учебник алгебры из простых на мой вкус. Из более продвинутых есть Aluffi, который охуенен.
А трюк вообще все умалчивают почему-то, кажется, сам придумал, а встречал всего пару раз.
Возможно, потому что если неправильно формулировать, то работает только над R и С из-за слов "всюду плотно" и "меры ноль". Но на самом деле мы утверждаем, что некоторый многочлен с коэффициентами из Z тождественно равен нулю (т.е. все коэффициенты равны нулю). А это никак не зависит от поля.
Аноним # OP  26/03/16 Суб 22:09:15  389170
>>389165
лучше наверное идти после 4 курса, чтоб вместе с магой учиться в ШАДЕ.
Аноним 26/03/16 Суб 22:16:19  389172
>>389170
а если я специалистишка?
Аноним # OP  26/03/16 Суб 22:33:30  389182
>>389172
не знаю, как там у специалистов с нагрузкой на 5, 6 курсе. Подозреваю, что несильно от магистрантов отличается. Я сам сейчас на 2 курсе маги, буду поступать в шад, думаю, что на 2-3 курсе все-таки рановато идти, хотя если ты суперботан - попробуй.
Аноним 26/03/16 Суб 22:47:07  389184
>>389182
а за курсы и их отличия не пояснишь? у них на сайте то написано, но все же
Аноним 26/03/16 Суб 23:04:30  389188
Реально ли поступить в ШАД,учась на физфаке МГУ?И много ли таких?
Аноним # OP  26/03/16 Суб 23:10:16  389190
14590230162660.jpg (191Кб, 747x753)
14590230162701.jpg (218Кб, 778x818)
>>389184
Могу пояснить со слов знакомого, который учится первый год в ШАДе. Учится на отделении Computer Science. Это направление считается более практическим, направление Анализ данных же - более теоретическое. Сейчас появилось еще третье - Биг дата.

На отделении CS в 1 семаке 2 основных курса: Дискран и Алгоритмы. Дискран ведет Райгородский, знакомый говорит, что предмет сложный, у него с трудом получалось набрать даже проходной балл в заданиях(правда, возможно это из-за того, что он начинал делать их в последний момент), всего 3 задания в семестре, в каждом надо набрать баллов больше порога, тогда задания сдано. Чтоб сдать предмет, как я понял, надо просто сдать все 3 задания, типа зачет/незачет. Примеры 2 и 3 задания пикрелейтед.
С Алгоритмами ситуация вроде попроще: их надо просто регулярно делать.

Помимо основных 2 курсов, есть еще курсы по выбору.
Про С++ сказал, что скучно объясняют.
Лингвистика прикольная, особо не напрягает, там надо решать задачки, типа перевести текст с древнего языка, не зная значения конкретных слов, но зная правила грамматики.
Теория информации, говорит, вроде тоже норм.
Теория игр хз.
Аноним 26/03/16 Суб 23:38:20  389191
>>389190
эх, раз оно более практическое, значит, наверно, попасть еще сложнее хотя и так сложно же
Аноним 26/03/16 Суб 23:41:14  389192
>>389191
Нет, сдаешь письменный экзамен, потом собеседование, потом, если ты проходишь, ты уже выбираешь направление. Можешь выбрать любое.
Аноним 26/03/16 Суб 23:51:07  389201
>>389192
спасибо тебе, анон :3 Желаю поступить на какое кстати сам хочешь?, а мне остается пару лет готовиться и потом, надеюсь, поступить
Аноним 26/03/16 Суб 23:56:13  389208
>>389190
>Биг дата
Что это за отделение?
Аноним # OP  27/03/16 Вск 00:05:36  389211
>>389208
https://yandexdataschool.ru/edu-process
Не знаю, оно в 2015 только открылось.

>>389201
И тебе удачи. Я уже пытался поступить 2 года назад, но не прошел. Надеюсь, в этом году получится. Насчет направления сам пока не знаю. Но склоняюсь больше к CS.

>>386842
Я, кстати, здесь проебался похоже. Необходимое условие для поступления - быть второкурсником(и выше) на момент начала учебы. Если ты сейчас 1 курс, то поступив в ШАД, и в сентябре приступив к занятиям там, ты уже будешь на 2 курсе универа.
Аноним 27/03/16 Вск 00:14:35  389213
>>389211
из какого ты вузика кстати, если не секрет?
Аноним # OP  27/03/16 Вск 00:20:47  389214
>>389213
физтех
Аноним 27/03/16 Вск 00:22:12  389215
>>389214
ФИВТ должно быть?
Аноним # OP  27/03/16 Вск 00:29:34  389218
>>389215
не, не ФИВТ, но то чем я занимаюсь, так или иначе связано с программированием и анализом данных, поэтому и хочу в ШАД
Аноним # OP  27/03/16 Вск 00:33:13  389219
>>389188
Реально
>21 человек из МГУ (14 с мехмата, 3 с ВМиК, 4 с физфака), 11 из МФTИ, а еще есть студенты из ВШЭ, МГТУ им. Баумана, МИФИ, УрГУ, МАИ, РУДН;
https://yandex.ru/blog/company/47442/?ncrnd=4244

>>389213
А ты откуда?
Аноним 27/03/16 Вск 01:12:57  389223
>>389219
бауманка
Аноним 27/03/16 Вск 18:06:40  389323
>>388998
Там exercise 1.3 смотри:

http://ocw.mit.edu/courses/electrical-engineering-and-computer-science/6-241j-dynamic-systems-and-control-spring-2011/assignments/MIT6_241JS11_assn01_sol.pdf
Аноним 27/03/16 Вск 18:28:04  389334
>>389014
> Где лучше мне почитать про хитрые методы взятия интегралов? У Фихтенгольцца?
https://www.coursera.org/learn/integration-calculus/home/welcome
Аноним 27/03/16 Вск 19:35:10  389346
http://yufeizhao.com/olympiad/det_eval_man.pdf
Аноним 27/03/16 Вск 19:36:05  389347
http://yufeizhao.com/olympiad/putnam_linear_algebra.pdf
Аноним 27/03/16 Вск 20:49:58  389350
>>389169
Если доказать что многочлен с коэффициентами из Z равен нулю, то это и значит что множество нигде не плотно(замкнуто по Зарисскому)
Аноним 30/03/16 Срд 17:30:48  389854
>>389073
Леша, ето ти?
Аноним 30/03/16 Срд 19:05:39  389869
А есть решение варианта от 7 июня 2015?
Аноним 30/03/16 Срд 22:40:15  389899
>>389869
Писал этот вариант год назад. 2 задача: мы выбираем базис в котором матрица имеет верхнетреугольный вид, на диагонали стоят собств значения, поскольку ранг матрицы - инвариант, то n значений на диагонали нули, но след матрицы также инвариантен относительно смены базиса, а стало быть n+1 собств значение равно следу
3 задача вообще в лоб решается, ответ 1
Аноним 30/03/16 Срд 23:26:35  389916
14593695957690.jpg (696Кб, 2560x1920)
>>389899
Меня больше интересует 5, 6,7 и 8 задачи. Вариант на пике.
Аноним 30/03/16 Срд 23:47:54  389917
>>389869
4 задача здесь >>389347 Problem 4
Аноним 30/03/16 Срд 23:49:32  389920
>>389917
4 задача это же изи. Всего лишь коммутатор посчитать. А вот 6 и 7 вообще не знаю как решать((
Аноним 31/03/16 Чтв 00:00:22  389926
>>389920
Не понял насчет коммутатора.
XY = lambda1X + mu1Y
YX = lambda2Y + mu2X
Вообщк говоря, lambda1 <> lambda2, mu1 <> mu2.
Как отсюда коммутируемость следует?
Аноним 31/03/16 Чтв 00:12:34  389928
14593723545990.jpg (222Кб, 1000x1777)
>>389926
Аноним 31/03/16 Чтв 00:24:12  389931
>>389928
Спасибо
Аноним 31/03/16 Чтв 00:38:16  389934
14593738967350.jpg (438Кб, 2048x1152)
>>389931
Только, я сейчас заметил, это не совсем полное решение( там с произведением матриц проблема, оно может быть равно нулю в некоторых случаях), а полное решение, можно получить как в Problem 3 >>389347
. Я сейчас даже посчитал(смотри пик, но там ошибка не мю е, мю У).
Аноним 31/03/16 Чтв 00:41:38  389935
14593740980650.jpg (44Кб, 500x502)
дайте решение 3й задачи от 31 мая
Аноним 31/03/16 Чтв 00:54:40  389937
>>389935
Разве там не просто расписываешь площадь сегмента, а дальше просто домножаешь на плотности распределения угловых координат A и B, а затем интегрируешь?
Аноним # OP  31/03/16 Чтв 00:57:19  389939
14593750397100.jpg (731Кб, 2560x1920)
>>389935
Аноним # OP  31/03/16 Чтв 01:00:02  389940
14593752029660.jpg (433Кб, 1536x2048)
14593752029751.jpg (512Кб, 2560x1440)
14593752029852.jpg (122Кб, 768x1024)
14593752029883.jpg (262Кб, 768x1024)
ОП на связи. Вот вам варианты за 14 год.
Аноним 31/03/16 Чтв 11:35:17  390032
Ребят,поясните ньюфагу:если сейчас учусь на 2 курсе ФФ,поступаю в ШАД,2 года там учусь,то можно ли будет потом подрабатывать в Яндексе и параллельно учиться в маге?И сколько вообще там платят,если работать на постоянной основе?
Аноним 31/03/16 Чтв 12:57:41  390045
14594182613230.png (47Кб, 1052x544)
>>389935
>>389939

Решать в лоб ШАДовские задачи противопоказано. То есть можно (это лучше чем не решать). Но у всех них есть красивое решение, и вы себя будете хорошо показывать на собеседовании тем, что не просто умеете применять формулы, но видите суть.

Конкретно в этой задаче трюк в том, чтобы добавить дополняющий сектор, тогда площадь двух секторов это площадь полукруга − площадь прямоугольного треугольника, среднее значение которой считается в уме.
Аноним 31/03/16 Чтв 13:10:38  390048
>>390032
Можно на стажировку устроиться, на 20-30 часов в неделю. Платят за это около 30к в месяц
Аноним 31/03/16 Чтв 13:39:06  390053
А стоит ли устраиваться на стажировку(Стажер-исследователь (Data Mining, Machine Learning)) перед поступлением в ШАД?
Аноним 31/03/16 Чтв 16:39:29  390113
>>389940
Спасибо, ОП, будем решать.
>>390045
Это красиво, правда. Хотя с интегрированием вроде тоже не очень замороченное, там элементарные штуки.

Вы тут все проглотили, что экзамен вне ДС будет онлайн. Или все из ДС, и вам похуй?
Аноним 31/03/16 Чтв 20:13:04  390207
>>389940
В варианте от 1-го июня не понял третью. Там получается, что A — обратимая матрица, то есть можно последовательностью элементарных преобразований привести ее к единичной (при этом ранг не изменится), тогда с хуев rank(E+A)=7? Или типа раз уже привели A к единичной, то давайте домножим первые две строки на -1, как раз получим E+A=7, а оттуда E-A=2?
Аноним 31/03/16 Чтв 21:05:16  390228
>>390207
Нет. Он равен 7 так как A кососиметрическая матрица, и (E-A)^T = E^T - A^T= E + A
Аноним 31/03/16 Чтв 21:27:00  390244
>>390228
Она не может быть кососимметрической, потому что у кососимметрической матрицы размера n*n для нечетных n det=0, то есть матрица вырождена. Тут же A — обратимая матрица.
Аноним 31/03/16 Чтв 21:51:01  390252
>>390244
Более того, у Винберга есть задача: доказать, что A — отражение, тогда и только тогда, когда A^2=E ( характеристика поля отлична от 2). Отражение записывается как диагональная матрица с +1 и -1. По условию задачи у нас должно быть две -1 на диагонали, так что ответ все-таки 2.
Аноним 31/03/16 Чтв 21:51:28  390253
>>390252
>>390228
Аноним 31/03/16 Чтв 21:54:14  390255
>>390252
>Отражение записывается как диагональная матрица с +1 и -1. По условию задачи у нас должно быть две -1 на диагонали, так что ответ все-таки 2.
Такая диагональная запись получится, если взять прямую сумму подпространств размерности 7 и 2 соответственно.

Аноним 01/04/16 Птн 08:35:50  390291
>>390048
А если на постоянной основе работать,то сколько будут платить?
Аноним 01/04/16 Птн 13:09:53  390319
>>390291
Точно не могу сказать. Слышал, что требования в яндексе серьезные, а вот зарплаты не очень. Ну 60-70к будешь получать наверное.
Аноним 01/04/16 Птн 23:09:00  390438
Ну что, готовы в понедельник тест решать?
Аноним 02/04/16 Суб 00:19:15  390444
>>390319
Спасибо,понял.
Аноним 02/04/16 Суб 01:08:44  390455
>>388887
почему мы действуем матрицей на вектор степеней иксов? откуда вообще это?
Аноним 02/04/16 Суб 07:31:09  390461
>>390455
Это стандартный приём по нахождению спектра циклической матрицы.
Аноним 02/04/16 Суб 13:20:26  390493
Хуйня для задротов, потеряете 2 года, будете получать раза в 2 меньше в Ындыксе.
Аноним 02/04/16 Суб 13:40:59  390494
>>390493
А кто сказал что мы в Ындекс после шада собрались?
Аноним 02/04/16 Суб 14:52:44  390508
А разве в этих задачах про окружности и вероястность не зашит парадокс Бертрана?
Аноним 02/04/16 Суб 15:20:13  390530
>>390438
А что в этом тесте будет?
Аноним 02/04/16 Суб 15:30:37  390533
>>390530
Ну в прошлые года там было десять задач, около 8 на математику и 2 на программирование/алгоритмы
Аноним 02/04/16 Суб 15:32:24  390534
>>390533
Ну а сколько нужно решить для того, что бы на письменный экзамен попасть, и как отправлять решения? И какого они качества эти задачи?
Аноним 02/04/16 Суб 16:24:43  390548
>>390534
Ну в прошлом году надо было решить минимум 8 из 10. Задачки простенькие достаточно. В этот раз на решение будет дано 5 часов, это все скорее нужно для того чтобы отсеять совсем тупых.
Аноним 02/04/16 Суб 16:44:38  390551
>>390548
А сколько времени уходит на проверку теста, а то вдруг я совсем тупой?
Аноним 02/04/16 Суб 17:59:08  390573
>>390548
А есть вариант?
Аноним 02/04/16 Суб 18:23:32  390576
>>390551
Если совсем тупой, то тебя не нужно пускать далее. Проблема теста в том, что можно ошибиться по глупости, и слететь из-за этого. Лучше всего его независимо прорешать дважды, затем сверить свои решения. В том году время не лимитировали, что уменьшало риски накосячить.
Аноним 02/04/16 Суб 18:29:56  390577
>>390576
Там нет лимита времени на тест?
Аноним 02/04/16 Суб 20:36:24  390591
>>390551
тест будет проводиться с 4 апреля по 10 мая. 11-12 мая все результаты объявят. так что до последнего ты не узнаешь, правильно ты решил или нет.
Аноним 02/04/16 Суб 20:38:56  390594
>>390576
Вообще да, мы с друзьями планируем зарегать анкету на одногруппника, который в шад не будет поступать, ему придет вариант, мы его прорешаем, потом свои анкеты зарегаем. там в вариантах, я думаю, максимум числа отличаться будут, сами задачи по сути скорее всего будут те же.
Аноним # OP  02/04/16 Суб 20:39:26  390595
>>390577
я же написал, на тест в этот раз 5 часов дается
Аноним # OP  02/04/16 Суб 20:47:15  390599
http://m2-ch.ru/un/res/209495.html
прошлый шад-тред
Аноним 02/04/16 Суб 22:14:41  390636
>>390594
Будь лаской выложи вариант онлайн теста для всех. Что бы анон не создавал для себя лишних фейко акков.
Аноним 02/04/16 Суб 23:22:10  390642
>>390508
Нет не зашит. Парадокс Бертрана про случайную хорду, вероятностное пространство которой можно задавать по разному. В этих задачах равновероятно выбираются точки на измеримом множестве, поэтому вероятностное пространство однозначно определено.
Аноним 03/04/16 Вск 00:53:50  390654
>>390636
нет его, в этом-то и дело
Аноним 03/04/16 Вск 01:41:15  390659
>>390654
Я имел ввиду после 4 апреля.
Аноним 03/04/16 Вск 01:44:41  390660
>>390659
Ок. Как бы потом шадовцы этот тред не нашли и не изменили условия потом, лол
Аноним 03/04/16 Вск 13:02:50  390702
14596777709880.gif (1277Кб, 280x200)
Бля, вы что, серьезно собираетесь жульничать для того, чтобы сдать предварительное онлайн-тестирование? Я правда не пытаюсь никого оскорбить, но если у вас возникают проблемы на данном этапе, то ни на письменном экзамене, ни уж тем более на собеседовании у вас нет ни единого шанса, сколько бы макулатуры вы с собой не взяли.
Аноним 03/04/16 Вск 15:54:44  390724
>>390702
в тесте можно ошибиться по глупости.
Аноним 03/04/16 Вск 22:19:14  390766
>>390702
Глупо приступать к заданию, без знания того что там вообще будет, а вдруг там один из вариантов письменного экзамена, к тому же там МНОГО вариантов, списать не получиться, поэтому это разведка, а не жульничество.
Аноним 04/04/16 Пнд 11:49:22  390821
В прошлом году были подобные треды, и там был длинный список магистратур на тему data science, может кто-то вкинуть?
Аноним 04/04/16 Пнд 13:15:56  390827
>>390821
>>390599
Аноним 04/04/16 Пнд 18:26:14  390874
>>390724
>>390766
На экзамене тоже можно ошибиться по глупости. Решайте вдумчивее, перепроверяйте.

мимо очень невнимательный анон
Аноним 04/04/16 Пнд 21:41:26  390937
Ну вбросьте уже тест ало. У меня комп сломался
Аноним 04/04/16 Пнд 23:01:09  390956
Что это вы тут затеяли, пидорашки? Черканул письмецо.
Аноним 04/04/16 Пнд 23:02:00  390958
>>390937
я через несколько недель сброшу может быть
Аноним 04/04/16 Пнд 23:11:00  390960
>>390958
через несколько дней
самофикс
Аноним 05/04/16 Втр 02:00:50  390985
>>389916
слушай, а в 6 задаче разве нельзя просто дважды продифференцировать по иксу и получить условие, что df/dx >= 1, которое верно по условию? и всё
Аноним 05/04/16 Втр 02:57:04  390988
>>390985
Ну в общем, если добавить слова, о том f<g определеныне интегралы F<G. То в общем да.
Аноним 05/04/16 Втр 09:51:10  391043
14598390702880.jpg (9Кб, 685x313)
>>386272 (OP)
Аноним 05/04/16 Втр 13:49:54  391091
>>390985
df/dx <= 1, конечно же
Аноним 05/04/16 Втр 20:42:11  391248
>>390988
Нет, все-таки нельзя, из того, что одна функция меньше другой не следует, что тоже верно для производных. Нельзя дифференцировать неравенства.
Аноним 05/04/16 Втр 20:48:18  391249
>>391248
Из того, что f<g , на отрезке a=<b,следует, что для определенных интегралов выполняется то же неравенство. Тут из неравенства производных следует неравенство функций.
Аноним 06/04/16 Срд 18:30:18  391420
>>389940
25 мая 2014, 1 задача
Вроде просто, но я затупок. Так что проверьте.
Просуммируем все строки в первой строке. То есть Первая строка станет состоять из лямбд. Далее элементарными преобразованиями занулим все значения первого столбца (кроме первой строки). Далее матрицу n-1 на n-1 приведём к верхнетреугольной форме (всгда возможно). В итоге у нас будет верхнетреугольная матрица с какой-то чушью на диагонали плюс лямбдой. Теперь просто стандартно ищем собственные значения, вычитая из нашей матрицы единичную, умноженную на х. Определитель верхнетреугольной матрицы равен произведению диагональных элементов, следовательно там есть член (лямбда - х). Приравниваем его нулю, следовательно х = лямбда. Чтд. Всё верно?
У меня какие сомнения: элементарные преобразования портят собственные числа? Знаю, что определитель они не портят. Вообще анон поясни, что они портят, а что нет.
Аноним 06/04/16 Срд 20:07:30  391438
>>391420
Чот ты СЛОЖНЫЙ. Умножь матрицу на вектор (1, 1, ..., 1). Элементарные преобразования — это все равно что умножать на элементарные матрицы, то есть композиция отображения с каким-то изоморфизмом (то есть сохраняется ранг, определитель не сохраняется).
Аноним 06/04/16 Срд 20:12:40  391439
>>391438
Ух ты ж...действительно, так ГОРАЗДО легче. Спасибо.
Аноним 06/04/16 Срд 20:21:07  391441
>>389940
Опять же 25 ма 2014, 5 задача - алгоритм
Просто считать как числа Фибоначчи нельзя, так как a_n может быть дохера большим, что не влезет в даблы, лонг лонг даблы, и вообще во всё. В общем а_n нельзя считать явно, так как оно разрастается, и память уже не О(1), а её приходится выделять динамически. Что делать?
Или же можно сказать, что a_n ограничена каким-то типом, и просто посчитать через три переменные a_n-2, a_n-1, a_n рекурсивно. На хуй не пошлют?
Аноним 06/04/16 Срд 20:38:31  391445
>>391441
вообще имхо нужно пользоваться рекурентной формулой для n-го числа Фибоначчи (там через производящие функции она хорошо выводится)
Ограничение по памяти есть, по времени нет (сложность формулы в том, что там всякие радикалы вида корня из 5 есть, я сейчас не помню, а выводить лень, но гуглится она легко или смотри там в учебнике Городенцева в 3-4 главах и я хз что делать с округлением, но мне кажется идея в этом)
Аноним 06/04/16 Срд 20:40:28  391447
>>386272 (OP)
а вообще для человека не особо читавшего тред можете сказать на что сейчас уделить внимание?
Есть норм подготовка в алгебре на уровне Винберга и норм знания с НМУ, но, смотрю на ШАДовские задачи, а там решения нужно вто просто додуматься но опыта/практики в этом нет.
Знающие аноны, из каких сборников задач в основном задачи на ШАДе-то, как я понял Putnam и Садовничий? Есть еще какие сборники? т.к. боюсь завалиться на теории графов/какой-нибудь ебнутом интеграле который легко возьмется и в кодинге
Аноним 06/04/16 Срд 23:12:13  391496
>>391441
>>391445
Тут фибоначчи как бе не причём. Мы же не складываем числа а приписываем их.
a
b
ba
bab
babba
babbabab
babbababbabba

Алгортитм можно такой сделать примерно так:
1. делаем рекурентную функцию, которая вычисляет количество цифр n_k в наибольшем числе a_k такое, что n_k < i
2. после этого уменьшаем i на это число (i -= n_k)
3. Повторяем шаги 1 и 2 пока k > 2
4. После этого находим i-ую цифру в числе ba
Аноним 06/04/16 Срд 23:44:02  391510
>>391496
Фибоначи нужны для вычисления длины. А ты уверен что задача решаема с заданным условием по памяти так, как там длина An может быть очень велико?
Аноним 07/04/16 Чтв 00:20:34  391512
>>391441
>>391445
>>391496
На шел решение еще лучше (точно нигде не нужно длинной арифметики, кроме операций с i).
Алгоритм:
Пусть l1,l2 количество цифр в a1, a2 соответственно.
Так как a_n (n>=4) состоит из чередования a1 a2 и a2 a1 a2 ( легко доказать по индукции), то следующий алгоритм, при n>=4, найдёт i цифру, при затратах O(1) на память:

do{
if (i<= 2l2+l1) return a2a1a2;
else i -= 2
l2+l1;
if (i<= l1+l2) return a1a2;
else i -= l1+l2
}while (1);
Аноним 07/04/16 Чтв 00:21:56  391513
>>391512
только двач съел квадратные скобки, там вовращается i-я цифра числа
Аноним 07/04/16 Чтв 01:16:48  391527
>>389940
А в 3 задаче от 25 мая. нужно в любом случае 2 log n + 1 вопросов задать. Или можно обойтись меньшим количеством?
Аноним 07/04/16 Чтв 07:49:50  391549
>>391527
Можно (и нужно) меньше
>>391512
Да вот не состоит. Нету там закономерности. Была бы закономерность -- можно была бы вывести точную формулу для числа фибоначчи вроде: xa1 + ya2. Но такой нету. Так что нихера по индукции не доказавыется. Поправь, если я не прав.
Аноним 07/04/16 Чтв 08:29:57  391556
>>391496
ой, там приписывание, сорян.
Я увидел что задано первые 2 числа и прочитал вполглаза думая что An складывается из An-1 +An-2
Аноним 07/04/16 Чтв 10:07:35  391566
>>391510
Обычно задачи таковы, что числа подающиеся на вход залезают в обычные типы, иначе об этом говорят особо. Поскольку на вход нам подаётся число i, которое влезает по такой логике в инт, а все числа в наших вычислениях меньше i, то нам нечего переживать.
Аноним 07/04/16 Чтв 10:09:35  391567
>>391512
Если бы анон был внимательным, то он бы увидел, что уже седьмое число не вписывается в его логику

bab ba bab ba bba

индукция анона соснула
Аноним 08/04/16 Птн 21:33:46  391871
>>386272 (OP)
Ну что аноны, прошла неделя с начала приема, кто как тест написал(по вашему мнению)?
Аноним 08/04/16 Птн 23:41:20  391884
>>390494
А куда,если не секрет?Очень хочу поступить в ШАД,но хз куда после него можно податься,что бы получать нормальные деньги.
Аноним 09/04/16 Суб 00:16:06  391888
>>391884
На пример вот: https://hh.ru/vacancy/16435417?query=machine%20learning и https://hh.ru/vacancy/16355526?query=machine%20learning
Аноним 09/04/16 Суб 11:11:15  391917
14601894754040.png (166Кб, 336x273)
>>391884
Куда угодно за границу, и быстрее, пока железный занавес опять не опустили.
Аноним 09/04/16 Суб 12:47:45  391926
Сап, двощ! Хочу поступить в ШАД, какие подводные камни?
Аноним 09/04/16 Суб 14:21:45  391952
>>391926
Письменный экзамен и собеседование
Аноним 09/04/16 Суб 21:19:46  392045
14602259864830.jpg (38Кб, 512x386)
Анон, как посчитать вероятность того, что две случайных величины лежат в какой-то области, если известно их совместное распределение (нормальное, величины коррелируют). Интересуют способы помимо взятия этого огромного двойного интеграла, с бесконечными пределами (из-за неограниченности области).
Аноним 09/04/16 Суб 21:56:19  392053
>>392045
Двачую вопрос.
Аноним 09/04/16 Суб 22:07:28  392056
И еще интересно, как кто решал задачу про нахождение числа подстановок, коммутирующих с заданной подстановкой. Естесственно, не полным перебором

>>392045-кун
Аноним 09/04/16 Суб 22:23:35  392058
>>392045
Если все не так плохо, то через афинное преобразование, делаешь замену переменных, приводишь к виду exp(1/2*(x^2+y^2)), а дальше симетрия.
Аноним 10/04/16 Вск 02:56:57  392089
>>392056
где такая задача в ШАДе?
в тесте в этом чтоли?
скиньте вообще задачи оттуда плиз
Аноним 10/04/16 Вск 10:28:08  392115
>>392089
Да, в тесте. В этом году тест посложнее вычислительно, чем в прошлом (который вообще в уме решался помимо комбинаторики), еще и с ограничением в 5 часов (видимо усложнили в связи с переносом письменного экзамена в онлайн для заочников).
Задачи такие были:
- Найти количество нулей на конце записи произведения 123...n
- Даны две системы векторов. Нужно найти размерности пересечения и суммы их линейных оболочек.
- Простая задача на формулу полной вероятности
- Дана подстановка на n элементах. Найти количество подстановок на n элементах, коммутирующих с заданной.
- Посчитать вероятность нахождения двумерной с.в. с известным распределением в заданной полуплоскости.
- Посчитать вероятность того, что последовательность из 6 чисел от 0 до 9 монотонно невозрастает.
- Площадь, ограниченная параметрической кривой
- Простая задача про условную сходимость знакочередующихся рядов
- Нарисован график гладкой функции на отрезке. Найти количество точек, в которых ее производная равна 1.
- Две даунских задачи на программирование, как всегда решающихся в лоб.
Аноним 10/04/16 Вск 12:52:08  392140
>>392056
Через действия и orbit-stabilizer formula решается:

http://math.stackexchange.com/questions/840008/finding-the-centralizer-of-a-permutation?rq=1
Аноним 10/04/16 Вск 14:22:17  392155
>>392115
Спасибо, анон. Я так понимаю, задача с коммутирующими перестановками самая сложная
Аноним 11/04/16 Пнд 19:52:17  392459
Всё же большой шанс ошибиться. Вот же обидно будет.
Аноним 11/04/16 Пнд 22:34:34  392507
>>392056
http://math.stackexchange.com/questions/85817/order-of-the-centralizer-of-a-permutation
вот формула готовая
Аноним 13/04/16 Срд 10:02:38  392774
>>392115
Задача про распределние полуплоскости решается как анон выше написал.

Делаешь замену переменных таких, что распределение становится нормальным со средним (0,0) и сигмой (1,1). После этого твоё распределение становится цилиндрически симметричным (зависит от расстояния). Поэтому граница полуплоскости характеризуется только расстоянием до центра r.
Можно повернуть координаты так, что граница полуплоскости станет вертикальна. После этого искомая вероятность = P(x > r)
Аноним 13/04/16 Срд 15:04:01  392829
>>392774
там такая жопа получается с коэффициентами, если так делать, одни радикалы лезут
Аноним 13/04/16 Срд 20:15:18  392899
>- Две даунских задачи на программирование, как всегда решающихся в лоб.
Ну не знаю. Последняя задача простой не показалась.
Аноним 13/04/16 Срд 20:17:57  392900
>>392899 ---> >>392115

Аноним 13/04/16 Срд 22:32:39  392929
>>392899
А что хоть за задача?
Аноним 14/04/16 Чтв 10:24:05  392995
>>392115
> Посчитать вероятность того, что последовательность из 6 чисел от 0 до 9 монотонно невозрастает.
Поясните, как посчитать количество невозрастающих последовательностей длины 6? Мне чет ниче в голову не приходит кроме динамического программирования: нарисовать таблицу 6 x 10 и заполнить ее.
Аноним 14/04/16 Чтв 10:36:01  392997
>>392995
Или в этом тесте как раз и предполагается, что надо код писать?
Аноним 14/04/16 Чтв 11:47:18  393003
>>392997
Я, например, именно так и сделал - написал программу, генерящую все подобные последовательности. Время уже поджимало из-за гребаной задачи про подстановки
Аноним 14/04/16 Чтв 11:52:35  393005
dsadsa
Аноним 14/04/16 Чтв 12:27:06  393017
>>392995
Насколько я понял, количество невозр посл-тей длины k на n числах равно
Цэ из n+k-1 по k-1
Аноним 14/04/16 Чтв 12:33:21  393018
Аноны, поясните что за хрень. В задаче про вероятность я поворачиваю базисные вектора и переношу начало координат на искомую прямую. Дабы у меня пределы интегрирования стали например -inf<x<inf, 0<y<inf но мой интеграл, который должен быть меньше единицы начинает расходится. Там надо на якобиан какой-нибудь домножить или не в этом дело?
Аноним 14/04/16 Чтв 13:17:17  393029
14606290373370.png (11Кб, 557x521)
>>392929
Find the largest rectangular area possible in a given histogram where the largest rectangle can be made of a number of contiguous bars. For simplicity, assume that all bars have same width and the width is 1 unit.

For example, consider the following histogram with 7 bars of heights {6, 2, 5, 4, 5, 2, 6}. The largest possible rectangle possible is 12 (see the below figure, the max area rectangle is highlighted in red)
Аноним 14/04/16 Чтв 20:01:57  393163
>>392056
коммутативна -- это когда ab = ba?
почему ответ -- не 1?
для любого i < n: пусть a(i) = xui, b(xui) = pizda. a(b(i)) тоже должно быть равно pizda, тогда нам подходит только такие b, что b(i) a^{-1}(pizda). Так для каждого i мы указали единственно возможный b(i).
Аноним 14/04/16 Чтв 20:03:58  393165
>>393163
*коммутирует, а не коммутативна
Аноним 14/04/16 Чтв 20:09:57  393167
14606537976450.jpg (23Кб, 849x83)
>>393163
>почему ответ -- не 1?
потому что вот ответ
Аноним 14/04/16 Чтв 20:16:43  393169
14606542034350.jpg (44Кб, 840x145)
14606542034361.jpg (33Кб, 853x197)
>>393167
а вот что кострикин по этому поводу пишет
Аноним 17/04/16 Вск 19:12:55  393801
котаны, а у кого какая программа ежедневной подготовки?

1. почитал Винберга-Куроша
2. порешал Демидовича-Виленкина
3. разобрал задачку с прошлых лет
5. съел плюшку
6. REPEAT
Аноним 18/04/16 Пнд 00:56:33  393910
>>393801
>5. съел плюшку
скурил плюшку
Аноним 18/04/16 Пнд 21:07:44  394014
>>393801
порешал пару вариантов из шада, посмотрел признаки сходимости рядов, прочитал в винберге про теорию групп.
Зачем чего-еще делать, это же не гос экзамен. Литературой пользоваться можно.
PS
и сколько плюшек в день ты потребляешь?
Аноним 18/04/16 Пнд 22:50:23  394065
>>392829
Там нет никакой жопы с радикалами. У тебя руки кривые. Задача устная.
Аноним 19/04/16 Втр 11:08:26  394135
>>386272 (OP)
Кто-нибудь знает, зачем в экзамен включают откровенные баяны и задачи в духе посчитайте интеграл? Это же стимулирует не повышение математической культуры, а надрачивание Демидовича и идиотских олимпиадок.
Аноним 19/04/16 Втр 12:06:24  394139
>>394135
там обычно не больше 1 такой задачи. думаю, просто проверяют знание интегралов
Аноним 19/04/16 Втр 13:08:11  394148
>>394139
На сайте шада есть экзамены за прошлые годы и там, к примеру, есть задача "вычислить интеграл (sin x)^8 dx от 0 до 2pi. Чет бомбануло от этой задачи. Нахуя мне это уметь делать?
Аноним 19/04/16 Втр 13:36:53  394151
>>394148
Ну, допустим, я догадался че надо делать. Расписал (sin x)^8 как [(e^{ix} - e^{-ix})/2i]^8, а дальше раскрыл через бином Ньютона. Но все равно задача какая-то дебильная.
Аноним 20/04/16 Срд 01:01:02  394364
>>394014
>Зачем чего-еще делать
затем что в ШАД требуют умения решать задачки, а это как я теперь понял нарабатывается только практикой. на дне открытых дверей сказали, что чем больше их решишь на экзамене, тем меньше вероятность что тебя попросят решить парочку на собеседовании.

на плюшки как и любые в-ва я забил ибо с какого-то момента стал ценить своё чистое состояние и те приходы которые меня накрывают без искусственных раздражителей. забить-то забил, но псевдоюмор остался.
Аноним 20/04/16 Срд 01:16:22  394366
>>394364
> в ШАД требуют умения решать задачки
как ты вообще 1 курс закончил без умения решать задачи?
Аноним 20/04/16 Срд 01:29:27  394367
>>394364
Намек был про то, что жиробас, а не провещества.
Аноним 20/04/16 Срд 02:33:20  394371
>>394366
лал, на первом курсе ПМ может и решают такие же задачки как в ШАД, у меня же на ИСиТ всё было крайне лайтово - три семестра вышки и семестр дискретки – и всё! всё что касалось алгебраических структур отсутствовало начисто ибо сдав все экзамены на отл я открыл первую лекцию НМУ и просто охуел
Аноним 20/04/16 Срд 02:36:32  394372
>>394367
да, я жиробас с ИМТ 17… ну и пофигу)
Аноним 20/04/16 Срд 12:21:09  394391
Пиздец какой-то, я поступал в 2009, будучи 3 курсником, не понимал алгебру и матстат и решал на очке, в офисе яндекса 7/10 задач. Не поступив устроился на 20к в лабу и успокоился.

В 2012, после диплома не прошел собес в яндекс, устроился на 85к и успокоился.

В 2016 не прошел на стажировку, положил хуй на собес по вакансии, в других местах предлагают 160-180к и я, видимо, опять расслаблюсь.
Аноним 20/04/16 Срд 12:39:00  394393
>>394391
Тут я, пожалуй, добавлю, что в 2015 ходил вольным слушателем на несколько курсов шада.

Поделюсь с вами впечатлением о курсах, на которые ходил или смотрел видео в шадовской системе для своих:

Воронцовское Маш. Обучение: ну хуй знает, стенфордский cs229 Andrew Ng лучше и приятнее.

Глубокое Обучение: так себе, стенфордский cs231n by Andrej Karpathy доходчивее и приятнее.

Машинный перевод: пиздец, дед с филфака рассказывает про правиловые системы, которые на ОТиПЛе делались с 60х годов.

Байесовские Методы и Граф Модели - два хардокорных курса, которые я не стал изучать, т.к. все это сложнее нейросетей (простых в изучении), дающих state-of-the-art результаты во многих областях.

Алгоритмы, распред.системы, nlp, comp/vision - хватает на работе либо дико лень на это тратить время.


Аноним 20/04/16 Срд 15:04:18  394417
>>394393
Спасибо за наводку на лекции Andrew Ng
Аноним 20/04/16 Срд 15:09:01  394418
>>394393
ты в 2015 в яндексе работал? Или как ты вольнослушателем ходил?
Аноним 20/04/16 Срд 16:01:30  394434
>>394418
Не стану палиться и скажу, что вольнослушатели - сотрудники, участники тренировок яндекс CV/ML тусовочек, студенты рилейтед вузов, у которых предметы в ШАДе проходят. Так же пропуск раньше мог сделать знакомый препод, если ему и тебе это надо.
Аноним 20/04/16 Срд 17:38:01  394469
>>394393
>>394391
Добра тебе, мил человек.
Аноним 21/04/16 Чтв 13:15:44  394572
>>390660
Уже нашли.
Аноним 21/04/16 Чтв 19:19:39  394628
>>394572
ты чтоли из ШАДа? вот скажи мне, зачем давать в тесте такие вычислительные задачи? какой в этом смысл? человек может просто по невнимательности ошибиться.
зачем давать задачу с количеством коммутирующих перестановок, для решения которой нужно знать теорию групп?
Аноним 21/04/16 Чтв 19:31:16  394629
>>394628
Чтобы ты экзамен завалил нахуй.
Аноним 21/04/16 Чтв 19:35:40  394630
>>394629
та они долбоебы просто в своем яндексе
Аноним 21/04/16 Чтв 23:53:14  394687
>>394628
>нужно знать теорию групп
Что не так со знанием теории групп?
Аноним 22/04/16 Птн 02:02:50  394698
>>394687
ее нет в программе шада
Аноним 22/04/16 Птн 12:00:39  394730
>>394698
>>394687
Подстановки.
Определение подстановки, четность подстановок. Произведение под-
становок, разложение подстановок в произведение транспозиций и независимых циклов.

Теория групп позволяет перевести это на язык действий, но по факту шад не превысил своей программы с задачами на коммутирующие перестановки.
Аноним 23/04/16 Суб 00:08:44  394842
антош,
это поможет чем?
http://ium.mccme.ru/IUM-video.html#Fall-2015:
Аноним 23/04/16 Суб 00:22:41  394843
>>394842
если только анализ-1. у Шабата максимум всратый курс, где чуть ли не на третьей лекции студенты когомологии групп считают. по алгебре смотри гарвардские лекции — они ближе к реальности. остальные курсы просто не покрываются шадом, поэтому ничем не помогут.
Аноним 24/04/16 Вск 19:05:12  395183
сколько проходной балл интересно будет
Аноним 25/04/16 Пнд 01:13:15  395318
>>394843
Что за гарвардские лекции и где их посмотреть?
Аноним 25/04/16 Пнд 02:18:08  395320
>>395318
https://www.extension.harvard.edu/open-learning-initiative/abstract-algebra

они по книжке Артина, можешь его навернуть заместо лекций
Аноним 25/04/16 Пнд 02:35:04  395322
>>395320
Спасибо!
Аноним 25/04/16 Пнд 10:44:13  395339
>>395320
У нас это на первом семестре первого курса всё проходилось, лул. При том что спбгу днище
Аноним 25/04/16 Пнд 11:19:15  395344
>>395339
>У нас это на первом семестре первого курса всё проходилос
так тут тоже курс algebra 1

>При том что спбгу днище
не сомневался
Аноним 26/04/16 Втр 21:19:24  395750
Поясните, зачем эта ШАД вообще нужна? Какие профиты?
Аноним 27/04/16 Срд 04:23:25  395801
аноны, как искать число подстановок, коммутирующих с данной?
Аноним 27/04/16 Срд 13:35:01  395827
>>395801
>>393167
Аноним 27/04/16 Срд 15:00:58  395833
>>395827
спасибо
Аноним 27/04/16 Срд 19:21:27  395884
>>395750
BUMP
Аноним 28/04/16 Чтв 00:13:16  395954
>>392115
в прошлом году все было намного проще, пиздец я мудак, был же шанс и время. гавно бьлядь.

- Посчитать вероятность нахождения двумерной с.в. с известным распределением в заданной полуплоскости.
- Посчитать вероятность того, что последовательность из 6 чисел от 0 до 9 монотонно невозрастает.

это че за темы хоть?
Аноним 28/04/16 Чтв 12:27:07  396040
>>395954
Первое -- двойной интеграл функции распределения по нужной полуплоскости.

Второе -- динамическое программирование на бумажке:
dp[n][c] = число монотонно невозрастающих последовательностей из n чисел от 0 до 9, заканчивающихся на c.

dp[1][c] = 1 для всех c, остальное вычисляется рекуррентно.

Ответ -- это сумма dp[6][0] + .. + dp[6][9] (число нужных последовательностей), делённая на 10^6 (число всех последовательности).
Аноним 28/04/16 Чтв 14:45:39  396086
>>396040
Проще же посчитать число монотонно возрастающих, а потом из единицы вычесть.

мимо
Аноним 28/04/16 Чтв 15:45:06  396109
>>396086
У тебя есть последовательности типа 121212, которые являются ни тем, ни другим
Аноним 28/04/16 Чтв 15:54:19  396114
>>396109
Все, понял.
Аноним 28/04/16 Чтв 18:59:12  396156
14618591528260.png (84Кб, 980x628)
>>389025
Чет бомбит. Это же тупо баян. Если знаешь - решишь, не знаешь - не решишь. Нахуя это включили?
Аноним 30/04/16 Суб 02:55:12  396345
14619741124490.png (49Кб, 1546x154)
Анон, а это всерьёз только через комплексные числа доказывается? Какие ещё есть способы?

http://mathkonspekt.blogspot.ru/2013/03/blog-post.html

где-то находил, что эта задачка есть вариация семнадцатой проблемы гильберта, но сейчас что-то это не гуглится

«Докажите, что многочлен с действительными коэффициентами, принимающий на действительной оси только положительные значения, может быть представлен в виде суммы квадратов многочленов с действительными коэффициентами.»
Аноним 30/04/16 Суб 08:24:58  396359
>>396345
Не знаю насчет комплексных чисел. ведь, блядь, это же квадратичные формы, там в лоб дана подсказка.
Аноним 30/04/16 Суб 13:29:53  396380
>>396345
Да, комплексные числа, но задача элементарная при этом.

>на действительной оси только положительные значения
многочлен с вещ. коэффициентами + нет действительных корней ==> все корни распадаются в пары комплексно-сопряжённых ==> многочлен есть произведение двух сопряжённых комплексных многочленов и старшего коэффициента исходного многочлена a0, который обязан быть вещественным и положительным (иначе бы на больших аргументах вылезало отрицательное значение).

Итак, исходный многочлен = a0(P+iQ)(P-iQ)=(sqrt(a0)P)^2+(sqrt(a0)Q)^2, где P и Q вещественные многочлены.


Что использовалось в док-ве:
https://en.wikipedia.org/wiki/Fundamental_theorem_of_algebra -- фундаментальная теорема "алгебры", о том, как любой комплексный многочлен представляется произведением.

https://en.wikipedia.org/wiki/Complex_conjugate_root_theorem (если для вещ. многочлена a+bi -- комплексный корень, то и a-bi -- тоже, теорема простая, можно доказать самостоятельно)
Аноним 30/04/16 Суб 19:07:44  396427
>>396380
Какого курса эта программа? Мимо 11кл
Аноним 30/04/16 Суб 20:51:06  396432
>>396427
В матшколах проходят чуть ли не с 8-ого класса. По обычным программам тех вузов -- первый семестр первого курса.
Аноним 01/05/16 Вск 00:45:28  396465
>>396432
Комплескный анализ на 1 курсе обычно не изучают
Аноним 01/05/16 Вск 00:59:53  396466
>>396465
У нас это было на алгебре. Док-во основной теоремы алгебры было более сложным, чем стандартное, но укладывающимся в знания первокура.

На анализе, разумеется, это было тупым применением теоремы Луивилля. Различные методы док-ва есть на странице англовики https://en.wikipedia.org/wiki/Fundamental_theorem_of_algebra#Proofs
Аноним 01/05/16 Вск 19:59:16  396585
Задачу про коммутирующие перестановки не поясните дауну
Аноним 01/05/16 Вск 19:59:36  396586
>>396585
поясните
хотфикс
Аноним 01/05/16 Вск 20:03:26  396588
>>396585
>>393167
Аноним 01/05/16 Вск 23:01:43  396620
>>396585
Имеется перестановка. Что выполняется, если другая перестанвка коммутирует с ней?

Она переводит циклы в циклы. Смотришь, куда переходит любой цикл, понимаешь, что коммутативность однозначно это определяет. И наоборот, если циклы переходят в циклы, коммутативность соблюдается.

Так что ответ -- кол-во способов перевести циклы в циклы. Ответ = Произведение [i=1..n] [c_i! * i^{c_i}].
n = длина перестановки, c_k = число циклов длины k.
Аноним 02/05/16 Пнд 05:50:43  396666
>>395884
HUIAMP
Аноним 02/05/16 Пнд 16:25:33  396796
>>395750
Учат разной попсе из машинного обучения. Отучившись, получаешь полезную строку в резюме.
Аноним 02/05/16 Пнд 19:22:38  396851
>>396380

Продолжаю разбираться…

Почему твое док-во и док-во по ссылке рассматривается для одной переменной? Или это все равносильно и для нескольких. Это из разряда очевидного или есть какие леммы/теоремы?

Вот эта теорема Complex conjugate root theorem (не знаю как она по-русски) она также на одну переменную. Здесь аналогично, как и в предыдущем случае?

Другой вопрос, почему многочлен раскладывается в $a0(P-iQ)*(P+iQ)$. Интересует происхождение коэффициента $a0$.
Аноним 03/05/16 Втр 00:16:01  396910
>>396851
>Почему твое док-во ... рассматривается для одной переменной?
Потому что задача вроде и была про многочлен от одной переменной, по выражению "на действительной оси" и отсутствию уточнений я сделал такой вывод.

>Почему ... док-во по ссылке рассматривается для одной переменной?
Основная теорема алгебры формулируется для многочлена одной переменной. Про сопряжённые корни то же самое.

>Почему многочлен раскладывается в $a0(P-iQ)*(P+iQ)$. Интересует происхождение коэффициента $a0$.
$a0$ берётся из основной теоремы алгебры, он же есть старший коэффициент многочлена. Из основной теоремы алгебры следует, что любой многочлен $P(x)=a_0(x-x_0)..(x-x_n)$, где $x_i$ -- корни $P(x)$, с учётом кратности
Аноним 03/05/16 Втр 22:51:52  397054
>>386272 (OP)
А о билайновском шаде что скажете, юные знатоки? А то мне время на ваши вузовские задачки тратить нерационально (наверное).
Аноним 04/05/16 Срд 00:20:56  397078
>>396345
>>396851
>«Докажите, что многочлен с действительными коэффициентами, принимающий на действительной оси только положительные значения, может быть представлен в виде суммы квадратов многочленов с действительными коэффициентами.»

Эта задача решается и для нескольких переменных тоже. Причём более элементарно, чем для одной.

Пусть многочлен -- не константа (иначе решать нечего).

Возьмём ненулевые x1, x2, .. xn, на которых P(x1, .. xn)=0. Если заменить любой из x_i на сопряжённый, это условие также соблюдается. Рассмотрим многочлен (X1-x1)((X1-conj(x1))+...+(Xn-xn)*(Xn-conj(xn)) от переменных X1..Xn. То, что он является вещественным многочленом, представимым суммой квадратов вещ. многочленов -- простое упражнение.

То, что исходный на него делится, следует из Nullstellensatz (strong form) https://en.wikipedia.org/wiki/Polynomial_ring#Hilbert.27s_Nullstellensatz. Делим и сводимся к той же задаче с меньшей степенью многочлена либо со степенью 0, для которой задача очевидна. Получаем, что исходный многочлен = произведение сумм квадратов вещ. многочленов -- значит тоже сумма квадратов вещ. многочленов.

Если что-то не понятно, за деталями обращаться. Обещаю, что все подробно не разобранное -- действительно простые упражнения.
Аноним 05/05/16 Чтв 02:01:10  397313
>>390252
а что насчет матрицы типа

0 1 0
1 0 0
0 0 1

наверняка аналогичная есть для размера 9*9
Аноним 05/05/16 Чтв 11:48:09  397340
>>397313
Тот анон припизделся, да. Все матрицы перестановок из циклов размера 2 и 1 будут нарушать эту задачу. А они, конечно, есть для матриц любого размера больше 1.
Аноним 05/05/16 Чтв 13:12:45  397348
>>397313
Это отражение, есличо. Возьми базис (1, 1, 0), (-1, 1, 0), (0, 0, 1). В нем получишь такую запись:

1 0 0
0 -1 0
0 0 1

>>397340
Я там потом добавил, что в нужном базисе матрица будет выглядеть с 1 и -1 на диагонали. Естественно, что в стандартном базисе так не получится.
Аноним 05/05/16 Чтв 13:17:31  397350
14624434515320.png (67Кб, 1086x499)
>>397340
Вот задача. Для перестановчной матрицы с циклом несложно видеть, что она будет отражать в плоскости (где, собственно, цикл). Бери подходящий базис и получай красивую диагональную запись.
Аноним 05/05/16 Чтв 14:19:42  397359
>>397350
>>397348
Тогда да. Пусть оператор R.

R(X)+X=R(R(X)+X) для всех X, так что простанство этих R(X)+X состоит только из собственных векторов R собственного значения 1.

R(X)-X=-R(R(X)-X) для всех X, так что пространство этих R(X)-X состоит только из собственных векторов R собственного значения -1.

Любые базисы этих двух пространств вместе образуют базис всего пространства, ведь X=((R(X)+X) - (R(X)-X))*1/2. 1/2 в поле есть, потому что характеристика поля не 2. Значит, любые пара базисов этих двух пространств -- как раз те, в которых оператор R имеет матрицу с 1 и -1.
Аноним 05/05/16 Чтв 18:31:49  397400
>>386316
А почему?
Аноним 05/05/16 Чтв 19:12:35  397416
>>397400
Потому что любое осмысленное программирование -- это просто перенесение матанов (и другой математики) в программу, большую часть которых человеку без знания матана даже не объяснишь. Стандартные вещи (вроде структур данных) людям, плохо понимающим математику, тоже крайне тяжело объяснить, не говоря об их способности делать модификации к стандартным алгоритмам.

На практике получаем, что предел мечтаний программиста без матанов -- это скриптики уровня таких, что скоро будут машинами писаться, а не людьми, очень плохое понимание базовых вещей, делающихся в этой профессии, а если и есть какое-то понимание, то обязательно крайне кривое, как бы деревенское (вот попробуйте объяснить человеку из деревни хотя бы сортировку, да, он будет очень туго и долго понимать, может быть годами, но главная фишка здесь не в этом, а в том, как он потом ОБЪЯСНЯТЬ понятое будет. Вот это действительно отпад)
Аноним 05/05/16 Чтв 19:16:07  397417
>>397416
А можешь привести пример хотя бы сферы программирования где эти самые матаны и линейки пригодятся? кроме очевидного геймдева
Просто я сейчас на первом курсе учусь и не совсем понимаю цели с которыми нам этот остопизделый анализ дают. Потому что дают очень сухо и без души. В отличие от той же алгебры и матлогики.
Аноним 05/05/16 Чтв 20:59:58  397441
>>397417
Раз ты в ШАД треде -- очевидный анализ данных и всякий компюьтер сайнс. Даже просто подгонка функции методом наименьших квадратов уже математика.
Всё, конечно же, зависит от твоих потребностей. Можешь сайты на wordpress'e делать, и считать себя программистом (но это неправда).
Аноним 05/05/16 Чтв 21:04:57  397442
>>397441
Понял. Звучит разумно, вроде
Я так понял, если я сейчас первокурсник, шансов попасть в ШАД нет?
Аноним 05/05/16 Чтв 21:11:10  397443
>>397442
У них ограничение по курсу. Надо быть 3 минимум. Но посмотри их задачи, почитай. Возможно заинтересует. И учиться будет интересней.
Аноним 05/05/16 Чтв 21:14:38  397444
>>397443
Бля. Ну что за ссанина. Чем вообще заниматься помимо учебы в вузе можно, если всё самое интересное начинается с третьего курса. Даже на работу берут только второкурсников-третьекурсников со средним баллом выше четырех. У меня горит жопа из-за подобной хуйни.
Аноним 05/05/16 Чтв 21:22:26  397445
>>397444
Спортом занимайся, или хобби каким.
Аноним 05/05/16 Чтв 21:26:41  397446
>>397445
Ну, я и так занимаюсь хобби.
У меня есть идея написать крипто-библиотеку какую-нибудь на сях. Но мне говорят что этого добра в интернете как говна.
Вообще не имею ни малейшего понятия чем заниматься. Как по жизни, так и во время обучения.
Тяжёлая жизнь первака, что тут скажешь.
Аноним 05/05/16 Чтв 21:39:03  397450
>>397446
Десятилетние школьники бабло от пейсбука за взлом инстаграммов получают, а ему видите ли заняться нечем, ибо первак. Доту катай.
Аноним 05/05/16 Чтв 21:42:18  397451
>>397450
>Доту катай
Да ну нахуй.
Просто хотелось бы чем-то действительно полезным заниматься. А то может в итоге оказаться что впустую тратил силы. А это очень неприятно осознавать
А про школьников с инстаграммами -- я почти уверен, что это просто одна большая случайность.
Аноним 05/05/16 Чтв 23:02:42  397471
>>397446
http://imperium.lenin.ru/~verbit/MATH/programma.html ну на, проверь, точно ли соответствуешь первокурснику.

На лето первокурснику в самую пору съездить в Дубну на математическую школу (дальше уже не смысла). http://www.mccme.ru/dubna/2016/ дедлайн на обычные заявки как раз до 10 мая, так что поднимай жопу и пиши, иначе возьмут только в случае оставшихся свободных мест.
Аноним 05/05/16 Чтв 23:13:21  397472
>>397471
Боюсь, я слишком нищ для такой хуйни.
Деньги дают мамка с папкой, а у них как раз последние полгода проблемы с работой. Считай, 30-40к за раз отстегнуть они мне не смогут
Наука -- это дорого, оказывается
Аноним 05/05/16 Чтв 23:19:32  397473
>>397472
Но оргвзнос 15 штук (включая питание и проживание) по данным сайта, к тому же иногда оплачивает университет, особенно 1-2-курсникам, если у него запросишь (сходи в деканат и проверь).

Это не так много. Платишь не столько за науку, сколько за определённый, более эффективный, способ дать себе в ней реальный толчок, не говоря уже о радости иметь вокруг реально заинтересованных людей. Заявку всё же подай, отказаться всегда можно, а год пропускать (если всё же захочешь во 2 курсе) как-то жалко.
Аноним 05/05/16 Чтв 23:22:42  397475
>>397473
Так ведь путешествие в Москвабад, туда и обратно, даже из Новосибирска это еще как минимум 20к
В сумме те самые 30-40к и получаются
Я, конечно, дойду до деканата и попробую разузнать там про это. И подам заявку. Но шансов на то, что я туда полечу, очень мало
Аноним 05/05/16 Чтв 23:39:43  397482
>>397475
Посмотрел цены на самолёт в июле, и правда дофига, от 16 тысяч. Хотя плацкарт на июль по данным tutu.ru -- около 6 тысяч, но там ехать 2 дня, пиздец.

Короче, давай так, если в универе не согласятся платить за самолёт. Я всё равно из буржуйской семьи, мне это всё задарма было. Если тебя берут, пересылаешь мне письмо, что взяли (до этого момента ты всё равно ничего не теряешь и можешь отказаться от участия), я скидываю на карточку денег, сколько нужно на поездку туда-обратно или покупаю электронный билет на нормальный самолёт. Если норм, фейкомыльцами обменяемся, присылай сначала своё мыло, дальше я тебе напишу (это чтобы левые чуваки отсюда не написали мне, впрочем, такие вряд ли сидят на /un/)
Аноним 05/05/16 Чтв 23:46:24  397484
>>397482
Ты уверен, что тебе это нужно?
uerrow@gmail.com
Аноним 05/05/16 Чтв 23:47:20  397485
>>397484
Да, считаю это своим долгом.
Аноним 05/05/16 Чтв 23:49:55  397487
>>397484
Написал с фейкопочты письмо-тест, проверь.
Аноним 06/05/16 Птн 08:14:46  397513
>>397471
Посмотрел я (не тот, которому отвечали) программу и охуел. Где-то так учат? Так надо? Не дохрена ли всего там намешано? Если человек хочет специализироваться, нахера ему столько лишнего, когда можно было в общих чертах?
Поясни, анон. Сам я не математик (по данной классификации я первокур).
Аноним 06/05/16 Птн 11:11:52  397534
>>397513
Кое-где так примерно и учат (НМУ). Кое-кто так учится сам.

Ну, ты вот если первокур по этой программе. По программе второго курса имеется два пункта:
> Векторные расслоения, связность, формула Гаусса-Бонне, классы Эйлера, Черна, Понтрягина, Штифеля-Уитни. Мультипликативность характера Черна. Классифицирующие пространства ("Характеристические Классы", Милнор и Сташеф).
>Дифференциальная геометрия. Связность Леви-Чивита, кривизна, алгебраическое и дифференциальное тождество Бьянки. Поля Киллинга. Кривизна Гаусса двумерного риманова многообразия. Клеточное разбиение пространства петель в терминах геодезических. Теория Морса на пространстве петель (по книге Милнора "Теория Морса" и Артура Бессе "Эйнштейновы Многообразия"). Главные расслоения и связности в них.

По сути, без них не сформулировать общую теорию относительности, и не понять ни на каком, кроме обывательского, уровня.
Аноним 06/05/16 Птн 11:50:47  397538
>>397534
Ух ты ж. Ну что, буду учить программу второго курса в режиме "что-то знаю, что-то понимаю".
Аноним 06/05/16 Птн 12:29:05  397543
>>397471
Мне кажется "аналитику данных" из этой программы нужен только 1 пункт, а еще теорвер матстат и слупы, которых там нет. Странная программа
Аноним 06/05/16 Птн 13:27:08  397553
>>397471
Во-первых, сам Вербицкий признал, что эта программа устарела. Вот его новая программа (она охватывает первые два курса):

http://verbit.ru/Job/HSE/Curriculum/all.txt

>>397543
Ну, многообразия (и соответствующий технический аппарат) могут пригодиться в том, чтобы снижать размерность данных, например. Но это серьезная наука, инженеру этого действительно не надо.
Аноним 06/05/16 Птн 21:17:29  397636
>>397553
Где же он такое признавал? То, что ты привёл, это сокращённая и переработанная программа, предназначенная для студентов матфака ВШЭ.
Аноним 06/05/16 Птн 21:27:38  397638
>>397543
Согласен, отсутствие теорвера -- КРАЙНЕ странно. Учитывая, что его понятия нужны и в физике тоже. Но вот что я заметил. Авот сначала пишет святые слова:

>Мне не кажется, что все области математики одинаково ценные; я уверен, что самоценности математика сама по себе не имеет. Иначе математика оказывается своего рода сложной интеллектуальной игрой, и мы оказываемся в области, обозначенной Германом Гессе ("Игра в бисер"), где никаких критериев нет вообще - кроме оценки профессионального сообщества. А профессиональное сообщество, что и скрывать, одновременно и коррумпировано, и разобщено. Профессиональное сообщество математиков не имеет единого критерия, а если бы и имело его, это было бы только хуже, наверное, потому что он был бы основан на невнятных властных играх по принципу ты почеши мне, а я почешу тебе, а ля академия наук.

И.. вот это поворот:

>Я думаю, что это не случайно. Математика утеряла общие критерии, потеряв общий контекст; в настоящий момент, гораздо меньше людей понимают, что происходит в науке в целом, чем 20 лет назад, и еще меньше, чем 40 лет назад. В условиях потери абстрактных критериев, единственно эффективным критерием становится утилитарный. Математика лишь постольку интересна, поскольку она связана со струнной теорией; это базовое предположение, которое я не хочу сейчас обсуждать. Релевантность для физики это единственный критерий, который у нас остался; а почти вся математика, относящаяся к физике, относится к струнной геометрии. Этот тезис хорошо подтверждается наблюдением, приведенным выше: (почти) все интересные идеи последних 20 лет связаны с физикой струн.

Кажется, речь просто о занятии математикой, как наукой. Созданием математики. Оно возможно и правда требуется сейчас только струнщикам. Теорвер-то это древняя наука. Так что да, похоже эта программа именно и только для математиков, которые хотят именно математикой заниматься.
Аноним 06/05/16 Птн 21:28:06  397639
>>397638
*Автор
Аноним 06/05/16 Птн 21:44:16  397644
>>397638
Мне кажется, я знаю что ответил бы вербит на претензию к отсутствию теорвера.
Типа, есть теория меры и дискретная математика. А все остальное в теорвере и так просто, студенты это сами изучат если им надо
Аноним 06/05/16 Птн 21:45:56  397645
>>397553
Блин, я имел в виду не первый пункт, а первый курс. Но и то он не весь нужен
Аноним 06/05/16 Птн 22:13:57  397647
>>397644
Ну, всё же теорвер имеет и независимую от них содержательность.
Вот, например: http://ium.mccme.ru/s10/probability.html
Эти теоремы не нужны? Охренительно нужны. Не математика? Да вот сложно сказать, что нет.
Аноним 07/05/16 Суб 00:22:29  397680
14625697495140.jpg (44Кб, 1035x172)
>>397647
АЗАЗА КАРТОФАНЧИК, ТИОРВЕР НИНУЖИН НИНУЖИИИИИИИН
Аноним 07/05/16 Суб 00:33:19  397684
Вброшу цитатку Ландау

Лев Давидович резко критиковал преподавание тематики на физфаках. Сохранилось его письмо ректору одного из московских вузов, в котором подробно излагаются взгляды на преподавание математики физикам:

"При всей важности математики для физиков, физики, как известно, нуждаются в считающей аналитической математике, математики же, по непонятной мне причине, подсовывают нам в качестве принудительнного ассортимента логические упражнения. В данной программе это прямо подчеркнуто в виде особого примечания в начале программы. Мне кажется, что давно пора обучать физиков тому, что они сами считают нужным для себя, а не спасать их души вопреки их собственному желанию. Мне не хочется дискутировать достойной средневековой схоластики мыслью, что путем изучения ненужных им вещей люди будто бы учаются логически мыслить.

Я категорически считаю, что из математики, изучаемой физиками, должны быть полностью изгнан всякие теоремы существования, слишком строгие доказательства и т. д. и т. п. Поэтому я не буду отдельно останавливаться на многочисленных пунктах Вашей программы, резко противоречащих этой точке зрения. Сделаю только некоторые дополнительные замечания.

Странное впечатление производит историческое ввдение. Само собой разумеется, что сообщение интересных исторических фактов может только сделать лекции более интересными. Но непонятно, зачем это рассматривать как пункт программы. Я надеюсь, что, по крайней мере, не имеется в виду спрашивать это на экзаменах. Векторный анализ располагается между кратными интегралами. Я не имею чего-либо против такого сочетания, однако, надеюсь, что оно не идет в ущерб крайне необходимому формальному знанию формул векторного анализа.

Программа по рядам особенно перегружена ненужными вещами, в которых тонут те немногие полезные сведения, которые совершенно необходимо знать о ряде и интеграле Фурье. Курс так называемой математической физики я считал бы правильным сделать факультативным. Нельзя требовать от физиков-экспериментаторов умения владеть этими вещами. Надо также отметить, что эта программа тоже сильно перегружена. Необходимость в курсе теории вероятностей довольно сомнительна. Физики и без того излагают то, что им нужно, в курсах квантовой механики и статистической физики. Во всяком случае, представленная программа переполнена бесполезностями. Таким образом, я считаю, что преподавание математики нуждается в серьезнейшей реформе".
Аноним 08/05/16 Вск 16:21:02  397822
>>397684
Ссылки на авторитет такие ссылки на авторитет.

Нет, я отчасти согласен с Ландау: программа по математике, по крайней мере, на моем факлуьтет, нуждается в сильном перереформировании. Большей части физиков действительно неинтересна и не нужна та математика, которой нас пичкают: формальные (но при этом устаревшие) выводы теорем и формул, упор на решение сотен примеров (и это в эпоху вольфрама и чсиленных решений).
Разумней было бы оставить Теорию в обзорном виде, сделать упор на важные для физики метса, а практику посвятить компьютерному вычислению математики. Это было бы во много раз полезней для большинства.
А что касается меньшинства, которые хотят заниматься серьзеной матфизикой и теорфизом - добро пожаловать на специально выделенные для вас места и программы, НМУ и матфак вышечки
Аноним 09/05/16 Пнд 15:26:57  397997
В онлайн-тесте на задачи про вероятность нужно давать ответы в процентах или в виде 0.1488? Почему-то нигде не написано.
Аноним 09/05/16 Пнд 16:48:59  398022
>>397997
там же описан формат вывода, все понятно вроде бы, не?
Аноним 10/05/16 Втр 20:27:48  398478
Ждём-с?
Аноним 10/05/16 Втр 21:36:37  398502
Ну чё, пацаны, когда там объявят результаты первого тура?
Аноним 10/05/16 Втр 21:59:45  398512
>>398502
Тем, кто прошел, уже пришли результаты.
Аноним 10/05/16 Втр 22:17:29  398516
>>398512
А как узнать результаты остальным?
Аноним 10/05/16 Втр 22:45:25  398522
>>398516
Вам (тем, кто не сдал) придут позже.
Аноним 10/05/16 Втр 23:32:27  398525
>>398512
Не пугай(
Аноним 11/05/16 Срд 16:52:20  398654
>>398512
Трололо?)
Аноним 11/05/16 Срд 18:45:29  398670
Чё там, ёпт?
Аноним 11/05/16 Срд 20:55:30  398733
>>396156
Откуда эти картинки, что за книжка?
Аноним 11/05/16 Срд 21:19:36  398746
>>398733
А че, прошел первый этап? Хочется не верить >>398512
Аноним 12/05/16 Чтв 08:40:38  398870
>>398746
Не, я в этом году не подготовился нихера и не писал. Вот хочу к следующему подготовиться, так что не выделывайся, а лучше подскажи автора и название той книжки.
Аноним 12/05/16 Чтв 13:35:38  398908
>>398733
L. Ridgway Scott: Numerical Analysis
Аноним 12/05/16 Чтв 14:26:07  398922
Про то что тем кто прошел уже пришли рез-ты - пиздеж.
Аноним # OP  12/05/16 Чтв 16:25:09  398947
Пришли результаты. 10/11 - fuck yeah!
Аноним 12/05/16 Чтв 17:11:35  398975
какой проходной балл?

5/11 не прошёл
Аноним 12/05/16 Чтв 17:23:30  398981
>>398947
Грац, ОП.
мимо 11/11
Аноним 12/05/16 Чтв 17:40:11  398988
Антоны, в задаче с определителем через операции над строками надо прийти к выводу, что определитель равен -+1 в зависимости от чётности/нечётности строк или я совсем дебил?
Аноним 12/05/16 Чтв 17:44:55  398989
лол 10 из 11, в одной проге время превышено на 0.078 с
Аноним 12/05/16 Чтв 17:45:48  398990
>>398975
от региона зависит наверное. 5/11 прошел
Аноним 12/05/16 Чтв 17:47:06  398991
>>398988
вроде того
у меня 2016х2016 ответ 1
Аноним 12/05/16 Чтв 17:49:21  398995
>>398991
Зачем так жить, похоже с минусом проебался. Ладно, в следующем году затащу.
Аноним 12/05/16 Чтв 17:52:08  398998
Отпишите ответы первого варианта, если у кого был, пожалуйста.
Аноним 12/05/16 Чтв 17:55:55  399002
>>398998
из верных
B: 0
C: 100
H: 3
+ 2 проги.

а у тебя?
Аноним 12/05/16 Чтв 17:58:29  399005
>>398988
я просто подсчитал определитель для маленьких матриц и продолжил результат на большие
Аноним 12/05/16 Чтв 17:59:47  399007
>>399005
просто написал 30 строк на С++. profit :D
Аноним 12/05/16 Чтв 18:01:15  399008
8/11 прошел. будьте добры правильные ответы F,H к 4 варианту, могу к другим задачам сказать, если кому интересно
Аноним 12/05/16 Чтв 18:01:39  399009
>>399007
мне кажется мой вариант быстрей )
Аноним 12/05/16 Чтв 18:02:40  399010
>>399008
Выложи свои ответы, будь добр.
Аноним 12/05/16 Чтв 18:07:22  399012
>>399008
Задачи лучше кинь, порешаем
Аноним 12/05/16 Чтв 18:09:27  399014
>>399012
http://rgho.st/6x2nZZGVf
пароль как обычно
Аноним 12/05/16 Чтв 18:09:32  399015
>>399010
на какие задачи
Аноним 12/05/16 Чтв 18:09:33  399016
Первый вариант правильные ответы (дополните плз):
A: 3 3
B: 0
C: 100
D: 0.25
E: 94770
F: Fail
G: Fail
H: 3
I: 1454.397
Аноним 12/05/16 Чтв 18:13:07  399018
>>399015
A B E I
Аноним 12/05/16 Чтв 18:16:10  399022
>>399018
>>399018
A: 0 5
B: 1
E: 278
I: 94247.7796

ответы на F,H имеются ?
Алексей 12/05/16 Чтв 18:16:39  399023
8/11 прошел. Был вариант 4
+A 0 5
+B 1
+C 6000
-D 0.338 (веротяно правильный ответ 0.291, но я считал как условную вероятность, т.к. вопрос сформулирован криво )
+E 278
-F 0.07 (накосячил с округлением, надо было до 3 знака)
+G 0
+H 1 2
+I 94247,7796
+J небольшая прога на питоне прошла все тесты
-K прошел 4 теста на пятом отвалилось
Аноним 12/05/16 Чтв 18:21:51  399026
>>399023
D 0.291 rly

я в F ответил 0.068, не зачли, у тебя сколько получается с правильным округлением?

K до 16 теста дошел, понятия не имею в чем косяк) писал на жаве
Аноним 12/05/16 Чтв 18:23:19  399027
>>399026
Тоже получилось 0.068, тоже не зачли.
Алексей 12/05/16 Чтв 18:25:26  399029
D честно говоря надо сформулировать вопрос более корректно, а то он не однозначен
F тоже 0.068 если тебе не зачли, то мне кажется у них косяк
Аноним 12/05/16 Чтв 18:54:13  399037
>>399002

только B верно.
Аноним 12/05/16 Чтв 19:28:45  399046
>>399029
Какие у тебя средние и дисперсия?
Алексей 12/05/16 Чтв 19:31:55  399047
M=(3,5)
E=((4,-2)
(-2,4))
Аноним 12/05/16 Чтв 19:36:28  399052
8/11 прошел
ошибся в D и G, лол
Аноним 12/05/16 Чтв 20:10:03  399066
4 вар F - какой правильный ответ? сам тоже отправил 0,068
Аноним 12/05/16 Чтв 20:11:24  399068
>>399014
а какой пароль обычно?
Аноним 12/05/16 Чтв 20:11:50  399070
>>399066
Как считал?
Аноним 12/05/16 Чтв 20:24:29  399079
>>399068
зелёный, съеби (2ch?)
Аноним 12/05/16 Чтв 20:42:10  399091
>>399068
2ch.hk
Аноним 12/05/16 Чтв 21:27:30  399103
>>399070
В матлабе на среднем из 10000000
Аноним 12/05/16 Чтв 21:29:44  399104
Подведены итоги первого этапа отбора в Школу анализа данных Яндекса. Количество решенных вами задач теста — 5.
Вердикт по каждой задаче вы можете посмотреть, вернувшись к странице тестирования: contest.yandex.ru/shad16/ (если на странице тестирования количество решенных задач отличается от приведенного выше, пожалуйста, напишите нам об этом).

К сожалению, этого результата недостаточно для прохождения на следующий этап отбора. Спасибо, что проявили интерес к Школе.

Как же грустно то.
Аноним 12/05/16 Чтв 21:33:53  399105
Аноны, школьник 11 класса, куда поступить чтоб учить все эти дата сайнс и то, что в заданиях было?
Аноним 12/05/16 Чтв 21:43:54  399110
>>399105
в университет
Аноним 12/05/16 Чтв 21:49:19  399111
>>399105
М(ФТИ) ФИВТ
Аноним 12/05/16 Чтв 21:50:21  399112
>>399105
ВШЭ ФКН
Аноним 12/05/16 Чтв 21:54:30  399115
>>399112
>>399111
Вот только я наберу 230-250
Аноним 12/05/16 Чтв 22:01:03  399116
>>399115
Тогда забудь про нормальное будущее.
Аноним 12/05/16 Чтв 22:08:02  399118
>>399116
Говори адрес, чмошник. Я тебе ебало разобью, ебанат, блядь. Совсем охуела мразь, нахуй ты срешь своим цинизмом в тред? Ты ебучее говно, нассал тебе на еблет.
Аноним 12/05/16 Чтв 22:27:23  399123
>>399118
Сорян, не буду больше.

>>399115
Присмотрись к СПбГУ, Политеху, там постарайся во всякие CS центры пробиться.
Аноним 12/05/16 Чтв 22:47:43  399126
>>399105
Никуда, универ не нужен. В /pr/ есть машин лернинг тред, там в шапке есть список литературы.
Аноним 12/05/16 Чтв 22:57:28  399130
>>399116
я с такими баллами и поступил на физтех 6 лет назад
Аноним 12/05/16 Чтв 23:44:25  399170
>>399126
Корка+армия. Алсо в книжках не совсем то, сейчас их читаю, и с преподами обычно учиться легче.
Аноним 12/05/16 Чтв 23:45:18  399173
>>399130
6 лет назад трава была зеленее.
Аноним 12/05/16 Чтв 23:53:54  399177
>>399130
я тоже поступил 6 лет назад, минимлаьный балл у нас на фак-те, вроде как, был 236, и то этот чувак был платником и дауном
Аноним 12/05/16 Чтв 23:55:57  399180
>>399177
ЛОЛ. я с 236 и поступил. но я не платник. и я был 4 с конца в списке.
Аноним 13/05/16 Птн 00:47:59  399196
>>399180
а, стоп, я же пмф. ты пми?
Аноним 13/05/16 Птн 03:27:14  399226
>>399196
нет, пмф. фак-и разные просто
Аноним 13/05/16 Птн 10:28:13  399258
>>399115
А нахуя тебе дата сайнс, если ты даже школьные предметы выучить не можешь?)
Аноним 13/05/16 Птн 10:56:50  399265
>>399226
На пми с 258 удалось поступить несколько лет назад. На ФИВТ рассчитывать при таких баллах можно только при наличии других заслуг, а вот ФАЛТ - вполне.

В тесте к задаче F четвертого варианта был у яндекса косяк все же.
Аноним 13/05/16 Птн 11:20:30  399272
>>399258
Из я учить не очень хочу, а дата сайнс хочу.
Аноним 13/05/16 Птн 12:00:55  399283
>>399265
как же я жалею, что не поступал на фивт, сейчас эта физика ебаная уже не интересна, хочется вкатится в разработку
Аноним # OP  13/05/16 Птн 13:57:51  399319
>>399283
Такие же мысли
Аноним 13/05/16 Птн 15:31:40  399349
>>399283
>>399319
Присоединяюсь.
Аноним 13/05/16 Птн 20:29:47  399563
>>399283
>>399319
>>399349
А пошли бы туда, сейчас бы скучали по физике.
Сам выбрал физику, теперь хочу вкатиться в ШАД. Вообще хорошо бы это объединить в своей жизни, ведь на стыке наук самое интересное.
Аноним 13/05/16 Птн 20:36:18  399570
>>399563
Мне понравилась твоя идея. Я ее немного иначе интерпретировал, походу мне нужно было поступить на физику, чтобы понять, что я не хочу ей заниматься, и благополучно свалить.
Аноним 13/05/16 Птн 21:37:57  399605
Господа, а в 4-ом варианте в последней задаче (про рекламу), решение за O(n²)? А то у меня по времени не прошло, писал на питоне. Вот думаю, из-за питона или алгоритмически не то.
Аноним 13/05/16 Птн 21:57:48  399609
>>399605
есть алгоритмы О(n)
Аноним 13/05/16 Птн 22:00:06  399611
>>399609
Например?
Аноним 13/05/16 Птн 22:26:27  399624
>>399611
http://lmgtfy.com/?q=histogram+largest+rectangle
Аноним 13/05/16 Птн 22:32:34  399628
>>399624
благодарю, анан. Главное знать, как гуглить, ага.
Аноним 13/05/16 Птн 22:44:29  399633
>>399624
А я бин поиском по второму массиву решил.
Аноним 13/05/16 Птн 22:58:50  399635
Ну и ещё вопрос по 4-ому варианту:
В задачи про вероятности как таки решать математически, без матлаба? Очевидно, что 2x₁-3x₂ не есть нормальное распределение и тут простым нахождением матожидания и дисперсии не отделаться.
Аноним 13/05/16 Птн 23:26:11  399639
>>399635
отправил ответ 0,068 - как раз через мат.ожидание и дисперсию считал. Сначала вердикт был отрицательный, но в пятницу (через день) появилось объявление жюри, что задача F перепроверена, и вердикт стал положительным.
Аноним 13/05/16 Птн 23:29:43  399642
>>399605
тоже писал на питоне. Завалили по времени на 36-ой проверке. Есть ли смысл писать "аппеляцию" через обратную связь?)
Аноним 13/05/16 Птн 23:37:19  399647
>>399639
Т.е. ты взял M[y] и D[y] и получил 0.068? Я получил 0.007, а сейчас вот думаю, а на каком основании вообще так можно делать?
Аноним 13/05/16 Птн 23:37:44  399648
>>399642
Нет, там O(n) есть и (красивое) O(nlog n)
Аноним 13/05/16 Птн 23:51:35  399652
>>399647
мат. ожидание ЧЕГО, дисперсия ЧЕГО? зачем? тебе нужно проинтегрировать плотность по той области пространства, которая удовлетворяет условиям, на этом задача исчерпывается
Аноним 13/05/16 Птн 23:58:56  399656
>>399652
угу, норм
Аноним 14/05/16 Суб 00:09:48  399663
>>399605
Вот мое решение. Идешь по массиву a.
Для a_i бин поиском находишь первое j такое, что b_j >= a_i. Понятно, что лучшая пара к a_i - это либо b_j, либо b_{j - 1}. Еще я здесь не расписал случай, когда a_i больше всего массива b.
Аноним 14/05/16 Суб 00:30:04  399670
>>399652
Дык, смотри. Линейна комбинация нормальной СВ это тоже нормальная СВ. Для этой СВ M = -9, D = 28. P{y > 4} = 1 - P{y < 4} = 1 - Ф((4 + 9)/5.291) = 1 - Ф(2.456) = 0.007.

Я, конечно, тупой, но ЧЯДНТ?
Аноним 14/05/16 Суб 09:53:38  399726
>>399670
Дисперсия другая.
DY = DY(X1,X2) = D(2X1-3X2) = 2^2DX1 + (-3)^2 DX2 + 22(-3)cov(X1,X2) = 44 +94-12(-2) = 16+36+24 = 76
P{y>4} = 1 - P{y<4} = 1 - [Ф0([4+9]/sqrt(76)) - Ф0(-беск)] = 1 - Ф0(1.491) - Ф0(беск) = 1 - 0.43189 - 0.5 = 0.06811
Аноним 14/05/16 Суб 10:00:50  399727
>>399652
Рискни здоровьем.
Какая там будет область? 2 x1 - 3 x2 > 4 - правильно? Осталось взять двойной интеграл совместной плотности распределения по указанной области...
Аноним 14/05/16 Суб 11:21:27  399732
>>399726
Спасибо, я таки знак потерял =(
Аноним 14/05/16 Суб 20:39:13  399887
>>397471
Что нужно знать, чтобы понимать лекции в этой школе? А то я 11-классник и всяких линейных алгебр и прочего не учил.
Может, стоит прочитать что-то перед поездкой?
Аноним 17/05/16 Втр 11:55:10  400936
>>399887
Достаточно быть олимпиадником в течение примерно 7 лет %)
Аноним # OP  19/05/16 Чтв 02:45:47  401883
Сфоткайте условия, кто 21 пойдет
Аноним 20/05/16 Птн 12:08:18  402378
какой был проходной на онлайн тестировании?
Аноним 20/05/16 Птн 14:41:59  402483
>>402378
Подозреваю, 7 или 8 баллов
Аноним 20/05/16 Птн 22:45:27  402747
>>402378
я прошел с 8.
Аноним 21/05/16 Суб 15:31:55  402917
Кто писал экзамен 21 мая? Сколько решили, какие задачи были?

От себя добавлю годноты: к олимпиадам готовлюсь по http://www.amazon.com/Art-Craft-Problem-Solving/dp/0471789011
Аноним 21/05/16 Суб 21:40:49  403093
>>402917
у меня сейчас сессия в разгаре, что мне делать, готовиться к письменной или сдавать экзамены, чтоб получить красный диплом?
Аноним 21/05/16 Суб 22:19:51  403111
>>403093
кто ж кроме тебя ответит на этот вопрос
Аноним # OP  22/05/16 Вск 02:10:52  403164
14638722529180.jpg (246Кб, 1920x1080)
Будьте ж людьми. Вкиньте условия 21 числа
Вариант от 21.05.2016 Аноним 22/05/16 Вск 02:36:32  403165
14638737931060.jpg (1133Кб, 2592x1936)

Вариант от 21.05.2016
Аноним 22/05/16 Вск 08:07:29  403185
>>403165
Как же бомбит от последней задачи. Это теорема Эрдеша, которая следует вот из этого
https://ru.wikipedia.org/wiki/%D0%A2%D0%B5%D0%BE%D1%80%D0%B5%D0%BC%D0%B0_%D0%94%D0%B8%D0%BB%D1%83%D0%BE%D1%80%D1%81%D0%B0

Просто НАХУЯ они включают в экзамен какие-то факты, которые ты либо знаешь либо не знаешь, а в условиях экзамена их невозможно доказать?
Аноним 22/05/16 Вск 08:27:38  403186
>>403185
А нет, я дебил просто.
Аноним 22/05/16 Вск 10:04:00  403212
>>403165
пиздец СЛОЖНА
Аноним 22/05/16 Вск 10:11:10  403213
>>403186
>>403185
Короче, вот
https://en.wikipedia.org/wiki/Erd%C5%91s%E2%80%93Szekeres_theorem
Бомбит чет.
Аноним 22/05/16 Вск 10:20:46  403215
>>403213
у меня бомбит от того что я приду и нихуя не решу
Аноним 22/05/16 Вск 10:27:32  403216
>>403215
А я не приду, потому что живу в Мухосранске и у меня в это время свои экзамены, лол.
Аноним 22/05/16 Вск 10:42:07  403217
14639029272560.png (77Кб, 1545x743)
>>403165
Решение 8
Аноним 22/05/16 Вск 11:12:57  403231
>>403216
мухосранские же онлайн пишут
Аноним 22/05/16 Вск 11:24:24  403234
>>403231
Ну это если ты не в Москву поступаешь, а я туда хотел.
Аноним 22/05/16 Вск 12:34:01  403263
>>403234
а на пары в Москву ты как собрался ездить ?
Аноним 22/05/16 Вск 12:38:25  403265
>>403263
Переехать собрался. Я сейчас заканчиваю 4 курс в Мухосранске.
Аноним 22/05/16 Вск 12:47:10  403272
>>403265
заебись тебе
Аноним 22/05/16 Вск 12:55:18  403276
>>403165
Говорят, в 5й таске алгоритм за O(N^2) есть, кто-нибудь в курсе, как такой реализовать?
Аноним 22/05/16 Вск 13:09:38  403292
>>403165
а есть вообще народ, кто решил задачу про окружности радиуса 1? среди моих знакомых даже imc'ишники не решили
Аноним 22/05/16 Вск 14:50:00  403350
>>403292
че за imc?
Аноним 22/05/16 Вск 15:00:58  403353
>>403350
а, все, нашел
Аноним 22/05/16 Вск 15:01:46  403354
>>403217
и откуда мы должны знать эту теорему дилуорса? ебаные шадовцы, нахер такое давать
Аноним 22/05/16 Вск 18:25:29  403431
>>403354
Ну 8 задача типа гробовая, чтоб задротам скучно не было.
Аноним 22/05/16 Вск 18:28:11  403433
товарищи, подскажите, пожалуйста, как в 6 эту сумму свернуть?
Аноним 22/05/16 Вск 20:34:03  403564
>>403354
Так-то это программа математического кружка стандартного
Вот тут третья ссылка))0)
http://bfy.tw/5tON
Аноним 22/05/16 Вск 21:38:36  403601
>>403564
Хуета, частный случай утверждения про цепи и антицепи. Здесь, по сути, можно сделать более сильное утверждение: взять нестрогий порядок вместо строгого и не требовать, чтобы числа были различными. Но ебучие олимпиадники как всегда придумывают даунские задачи, демонстрирующие СМЕКАЛОЧКУ составляющего и не демонстрирующие никаких важных математических идей.
Аноним 22/05/16 Вск 21:58:47  403619
>>403165
Помогите решить дауну. Вот первая. С ходу 2 pi k / n, а так же все экспоненты, так как к нулю стремится. Что вообще надо?
Аноним 22/05/16 Вск 22:00:02  403620
>>386272 (OP)
А что это за школа? Типа маги после, напрмиер, ВМКовской баки?
Аноним 22/05/16 Вск 22:03:17  403623
И зачем туда идут? Для чего? В чем профит от их сертификата?

Там конкуренция большая?

Если я физфаковец, но захочу пойти в айти, имеет смысл начинать оттуда? Опыта в айти нету, а там, думаю, сотни студентов с ПИ/ВМК/ПМ и прочих околоайтишных факов, у которых знаний и опыта раз в 5, 15, 40 больше, чем у меня, всякие джуниоры и сеньоры из АЙ-О-ЭС СТАРТАПОВ
Аноним 22/05/16 Вск 22:33:24  403635
>>403619
x должен быть конечным и не зависящим от n.
так что думаю подходят только 2pik
Аноним 23/05/16 Пнд 01:52:48  403753
>>403623
я сам из околофизики, бро. если тебе интересно машинное обучение - начни, уйти то всегда сможешь
Аноним 23/05/16 Пнд 08:49:46  403793
>>403623
больше дела, меньше пиздежа.
Аноним 23/05/16 Пнд 14:43:34  404062
Пацаны, у меня тут встал вопрос.
Скажем, существует бесконечномерное евклидово пространство многочленов над полем R.
Как в таком случае можно задать скалярное произведение векторов, чтобы разложение функции в ряд Маклорена было как раз-таки разложением по базису данного пространства?
Аноним 23/05/16 Пнд 15:23:50  404077
>>404062
Не понял вопроса. Ты раскладываешь в ряд Тейлора, там у тебя есть x^n/n!, бери их в качестве базиса и получай разложение по базису. Зачем тебе скалярное произведение?
Аноним 23/05/16 Пнд 15:26:36  404081
>>404077
Меня интересует вопрос такого рода:
Есть у нас векторное пространство V = <x^n/n | N э n>. Задача -- сконструировать скалярное произведение для векторов таким образом, чтобы разложение в ряд Маклорена стало разложением по базису данного пространства.
Я сам это придумал, не спрашивайте зачем
Аноним 23/05/16 Пнд 15:31:33  404087
>>404081
Пространство над R, если что
Аноним 23/05/16 Пнд 15:34:26  404092
>>404081
Это пространство содержит только многочлены, а других функций не содержит. Зачем тебе раскладывать многочлены в ряд?
Аноним 23/05/16 Пнд 15:36:07  404094
>>404092
Существуют же функции, которые раскладываются в ряд Маклорена, но при этом не являются многочленами. Та же e^x, например. То есть, существует бесконечный кортеж, который описывает e^x в данном пространстве. Значит, существуют и другие, для других функций.
Аноним 23/05/16 Пнд 15:41:33  404097
>>404094
Бля ты сказал, что V - это пространство многочленов, а не всех функций, которые раскладываются в ряд.
Аноним 23/05/16 Пнд 15:48:05  404098
14640076852760.png (31Кб, 873x191)
>>404097
Это бесконечномерное пространство многочленов.
Пусть a=0. Тогда функция f в данном пространстве будет иметь вот такое разложение. (x - a)^k -- базис, а f^(k)(a)/k! -- коэффициенты. Понимаешь?
Аноним 23/05/16 Пнд 15:49:56  404101
>>404098
(x - a)^k / k! -- базис, f^(k)(a) -- коэффициенты.
Быстрофикс
- - 23/05/16 Пнд 23:22:15  404491
а какого числа следующий экз? Письмо пофигачил нечаянно
Аноним 24/05/16 Втр 01:27:03  404544
>>404491
28 и 4
Аноним 24/05/16 Втр 01:27:32  404545
>>404491
28 мая
Аноним 24/05/16 Втр 14:20:46  404687
ОП, хуй,^H^H^H^H как у тебя дела?

будешь писать онлайн или очно?

все задачи прошлых лет прорешал?

всю ли теорию задрочил?
Аноним 24/05/16 Втр 14:27:05  404691
Кто-нибудь решил 7ую задачу с матрицами?
Аноним 24/05/16 Втр 16:06:13  404761
В 6 задаче про робота ответ случаем не ноль?
Аноним 24/05/16 Втр 16:27:24  404774
>>404761
Не ноль, очевидно же, что матожидание положительной случайной величины, которая может принять значение 4 и 6 (например, возьмите n = 6, тогда модуль разности может быть от 0 до 6), не может быть равна 0
Аноним 24/05/16 Втр 16:41:30  404778
>>404761
По-моему ответ должен выглядеть как-то так:
M(|X1-X2|) = sign(X1-X2) M(X1-X2) = sign(2X1-n) M(2X1-n) = sign(2X1-n) n * (2^(n) - 1)

Есть у кого другие варианты?
Аноним # OP  24/05/16 Втр 19:06:30  404887
>>404687
28 пойду. Нет, далеко не все прорешал, из того что хотел. В данный момент дрочу линал и алгоритмы, ибо с остальным проблем нет
Аноним 24/05/16 Втр 20:02:36  404908
>>404778
как это ты так интересно сигнум вынес, и просто посчитал мат ожидание?
учитывая, что X1 принимает значения от 0 до n, то количество минус единиц и единиц у тебя одинаковое, что обратит последнюю запись в ноль
или я что-то не так понял?
Аноним 24/05/16 Втр 20:02:50  404910
14641093701410.png (85Кб, 643x456)
14641093701421.png (108Кб, 684x426)
14641093701432.png (118Кб, 642x449)
>>399635
>>399647
Как-то так
Аноним 24/05/16 Втр 20:03:53  404911
>>404691

1) Нужно доказать, что uu^T * J - матрица с линейно зависимыми строками(все начиная со второй могут получиться умножением числа на первую строку)
2) умножение на число B картины не меняет
3) Воспользуемся http://portal.tpu.ru/SHARED/k/KONVAL/Sites/Russian_sites/2/05.htm - правило 8

Ответ: 1
Аноним 24/05/16 Втр 20:07:40  404912
а в мухосрансках экзамены легче будут, как думаете?
Аноним 24/05/16 Втр 20:21:25  404924
>>404778
>>404778
Вероятностное пространство задачи -- множество векторов длины n из 0 и 1, составленных по схеме Бернулли. Я рассмотрел случайную величину «разность между количеством 0 и 1 в векторе» и высчитал ее матожидание как сумму произведений значения величины (натуральные числа от 0 до n) на его вероятность. Вероятность нашел через вероятность k единиц в схеме испытаний Бернулли. Получилась сумма с биномиальными коэффициентами, деленная на 2^(n-1). До компактного не довел, ибо на знаю как посчитать сумму биномиальных коэффициентов C(n,k) при к от 0 до n/2.
Аноним 24/05/16 Втр 20:34:40  404943
>>404911
Анон, спасибо тебе огромное! Проверить на линейную зависимость в голову не приходило
Аноним 24/05/16 Втр 20:37:32  404951
>>404924
У меня в 6 задаче матожидание 1 при четном n. При нечетном считать лень.
Аноним 24/05/16 Втр 20:40:25  404954
>>404924
Я записал как рекуррентную формулу. Это не считается за компактную?
Аноним 24/05/16 Втр 20:41:12  404957
>>404954
Сам как думаешь?
Аноним 24/05/16 Втр 20:42:06  404959
>>404951
оно не может быть равным единице. Подставь к примеру n = 4 и не получишь мат ожидание = 1
Аноним 24/05/16 Втр 20:44:38  404962
>>404951
Не может быть такого. При каждом чётном броске матожидание не изменяется: шансы, что число уменьшится или увеличиться на 1, равны. Но при нечётном броске число 0 всегда увеличивается на 1, поэтому мат ожидание увеличивается на вероятность нуля при данном количестве бросков.

1 бросок: 1
2 бросок: 1
3 бросок: 1+1/2
4 бросок: 1+1/2
и т.д.
Аноним 24/05/16 Втр 20:44:45  404963
>>404959
куда подставить?
На первый взгляд, матожидание 1 для разности вполне логично, ибо переходы на черную и белую клетки равновероятны на каждом шаге.
Аноним 24/05/16 Втр 20:46:26  404966
>>404924
пришел к такому же решению, как свернуть гребаную сумму, кто-нибудь знает?
Аноним 24/05/16 Втр 20:47:45  404970
>>404966
Заметь, что 2^n = сумма C(n,k), k=0..n
Аноним 24/05/16 Втр 20:48:06  404971
>>404963
ну если ты утверждаешь, что для любого четного n, мат ожидание у тебя равно 1. так просто на примере проверь. возьми n = 4, и посчитай свое мат ожидание в лоб по известной формуле. и я тебя заверяю, единицу ты не получишь
Аноним 24/05/16 Втр 20:51:56  404974
>>404970
да, это правда. отсюда, продиффиренцировав, я могу получить значение суммы kC(n,k), k = 0...n
но как получить из этого сумму k
C(n,k) , k = 0..n/2?
Аноним 24/05/16 Втр 20:57:30  404986
>>404974
сумма С(n,k), k=0..n/2 равна 2^(n-1), ибо БК симметричны относительно n/2.
А каким образом ты «дифференцируешь» дискретные объекты, я не понял. Проверяющий тоже не поймет, имхо.
Аноним 24/05/16 Втр 21:02:18  404994
>>404951
Спасибо за замечание. Я был не прав насчет 1 для четных n. Видимо, криво свернул сумму.
Аноним 24/05/16 Втр 21:05:21  404999
>>404986
просто дело в том, что заявленное мат ожидание - не просто сумма С(n,k) k = 0...n\2, а скорее
k C(n,k) , k = 0...n\2
А дифференцирование просто проворачивается:
(1+x)^n = sum(C(n,k)x^k) - это функциональный ряд, который можно дифференцировать по x
отсюда мы можем получить sum(k
C(n,k)) = n*(2^(n-1)), k = 0...n
как отсюда можно получить данную сумму но для k = 0...n/2?
Аноним 24/05/16 Втр 21:35:05  405019
>>404986
>А каким образом ты «дифференцируешь» дискретные объекты, я не понял.
"Конкретную математику" Кнута полистай. Много полезных штук найдёшь.
Аноним 25/05/16 Срд 10:45:05  405142
>>404911
Может я чего то не понимаю. Давайте рассмотри матрицу ((0,1),(-1,0)), вектор (1,1) и число бета так и обозначим. В вашем решении вы никак не используете то. что матрица кососимметрична, кроме этого 8 свойство в данной ситуации не подходит, т.к. единичная матрица прибавляет только к 1 элементу каждой строки 1. в моем примере в итоге получим результат 2-бета что не равно 1
Аноним 25/05/16 Срд 11:23:15  405147
>>405142
Я не тот анон, но отвечу.
Кососимметричность матрицы используется чтобы доказать линейную зависимость строк матрицы uu^T*J.
Можно свойство 8 применить к каждой строке по отдельности, тогда всё кроме единицы сократится.
А в вашем примере определитель тоже получается = 1, перепроверьте.
Аноним 25/05/16 Срд 11:48:49  405151
>>404924
А поясните пожалуйста, почему сумму считаем до n/2?
Аноним 25/05/16 Срд 12:11:14  405162
>>405147
линейная зависимость строк указанной матрицы следует из линейной зависимости строк uu^T (т.к. её ранг=1), а не из кососимметричности

определитель действительно получился равным 1.

В моем примере если взять вектор (1,2), то не понимаю как вы примените свойство 8
т.к. надо будет найти определитель ((-2b+1,b),(-4b,2b+1)).
определитель действительно будет равен 1, но на мой взгляд данное обоснование не проходит.
Аноним 25/05/16 Срд 12:31:32  405171
>>405162
Например вот так. Разобьем сначала по свойству 8 матрицу по первой строке, тогда искомый определитель равен сумме определителей ((-2b,b),(-4b,2b+1)) и ((1,0),(-4b,2b+1)). Потом каждую из двух этих матриц разобъем по второй строке. Получим сумму уже из четырёх определителей ((-2b,b),(-4b,2b)); ((-2b,b),(0,1)); ((1,0),(-4b,2b)); ((1,0), (0,1)). Первый определитель = 0, последний = 1, а второй и третий сокращаются: -2b + 2b, кососимметричность наверняка пригодится где-то здесь.
Может быть даже проще можно доказать, не знаю
Аноним 25/05/16 Срд 15:45:26  405241
>>405151
Потому что в выражение вероятности события «разность черных и белых равна k» входит C( (n+k)/2, n ).
Аноним 25/05/16 Срд 16:41:46  405271
>>405241
ну вообще все мат ожидание запишется как
sum((1/2)^nC^i_n|n-2i|) но данный ряд симметричен поэтому в реале надо посчитать половину этой суммы.
Аноним 26/05/16 Чтв 10:23:25  405724
Ребят, я нашёл как решается 6 задача(+ там как упростить сумму биноминальных коэффициентов С(n,k) ):

http://math.stackexchange.com/questions/390655/expected-value-of-h-t-in-n-coin-flips

А вообще - это оказывается типовая задача:

http://mathworld.wolfram.com/RandomWalk1-Dimensional.html
Аноним 26/05/16 Чтв 15:17:27  406147
>>405724
молодчинка
Аноним 26/05/16 Чтв 17:19:16  406258
>>403165

Кто 2 решил? Не даётся: я вывел, что
характеристический многочлен P должен раскладываться на произведение Q(x^2 + (3/2)x+ 7/8) , так как полином должен содержать сопряжённый корень в задаче.

А что дальше?
Аноним 26/05/16 Чтв 22:57:28  406809
>>406258
http://dxdy.ru/topic108732.html
уже все решили
Аноним 26/05/16 Чтв 23:10:20  406819
>>403165
Мне одному показалось, что третья задача какая-то подозрительно легкая?
Аноним 27/05/16 Птн 03:40:42  406900
>>406819
1 и 3, чтобы случайные не ушли обиженными. Я только их и решил. Ну и 5 ещё попытался.

Мимоменеджмент
Аноним 27/05/16 Птн 03:42:59  406902
>>406258
ну я делал так. от противного пусть ламбда соб значение. тогда det(A-lambdaE)=0, а значит существуют две линейно зависимые строчки k и m. Рассмотрим (... , akk-lambda, ..., akm...) и
( ... , amk, ... , amm - lambda, ...)
они л/з , следовательно сущ нетривиальные c . d такие что:
c(akk-lambda) + damk =0
cakm+d(amm - lambda) = 0 . что бы решение было нужно что бы определитель был равен нулю, но такого быть не может , так как все aij целые lambda комплексная.
Аноним 27/05/16 Птн 10:47:25  407002
>>406900
А можно для совсем тупых - как 3 решать? С вероятностями туго
Аноним 27/05/16 Птн 11:52:37  407037
>>406902

тогда по вашему решению матрица
(1 -4)
(1 1)
Не имеет комплексных собственных чисел, но это не так - проверьте
Аноним 27/05/16 Птн 12:18:34  407082
>>406902
Ты так и написал, потому, что лямбда комплексная? Ты был в шаге от правильного решения(
Аноним 27/05/16 Птн 13:33:03  407157
>>407002
ботай функцию распределения суммы случайных величин
Аноним 27/05/16 Птн 15:06:28  407369
>>407157
зачем она в третьей задаче?
Аноним 27/05/16 Птн 15:06:56  407371
>>407369
а, понял, не так прочел условие
Аноним 27/05/16 Птн 16:21:03  407513
>>407002
Пусть S1 - площадь мишени, S2 - полщадь мишени вместе с рамкой шириной в величину случайного отклонения поля - 0,1. вероятность попадания в мишень p = S1/S2. S1 = 6, S2 = 7.04 => p = 0.8522..
Аноним 27/05/16 Птн 16:29:59  407525
>>405147
>>Кососимметричность матрицы используется
>>чтобы доказать линейную зависимость строк
>>матрицы uu^TJ
То ли я в чем-то туплю, то ли кососимметричность нужна только для запутывания.
Ибо ранг матрицы uu^T равен единице (по теореме о ранге произведения), и по этой же теореме ранг uu^T
J равен единице, т.е. все строки пропорциональны первой строке.
Аноним 27/05/16 Птн 16:33:33  407529
>>407513
Это было бы верно, если бы распределение координаты попадания было бы равномерное.
Аноним 27/05/16 Птн 16:46:21  407567
>>407513
маму твоя шатал, пол-часа искал у тебя косяк.
Нашёл.
Правильный ответ 0.954.
Сасай лалка.
Аноним 27/05/16 Птн 17:10:12  407615
>>407525
>ранг матрицы uu^T равен единице (по теореме о ранге произведения)
разве? я думал, там ранг 1, потому что можно указать алгорит, зануляющий все строки кроме первой эл. пр-ми
Аноним 27/05/16 Птн 17:14:32  407621
>>407615
Это как бы эквивалентные вещи еси че
Аноним 27/05/16 Птн 17:21:01  407635
>>404687
5 лет назад универ закончил, линала и матана маму рот ебал. Варианты 2012 и этого года вселяют радость, вариант 2013 года вселяет уныние
Аноним 27/05/16 Птн 17:33:03  407666
>>407529
А что, там не равномерное распределение? Владелец листочка даже аккуратно подписал "равном. распр.", как этому не верить?
Аноним 27/05/16 Птн 17:41:00  407691
>>403292
Докажем, что функция гармоническая.
Известно свойство среднего: Если функция u гармонична в некотором шаре B(x_0) с центром в точке x_0, то её значение в точке x_0 равно её среднему значению по границе этого шара. Обратно, любая непрерывная функция, обладающая свойством среднего для всех шаров, лежащих в некоторой области, является в этой области гармонической.
Ок, доказали, что наша функция гармоническая. Дальше изи. По теореме Лиувилля. Гармоническая функция, определённая на R^n и ограниченная сверху или снизу, постоянна.
Вот и все.
Аноним 27/05/16 Птн 17:46:45  407701
>>407567
И где ошибка?
Аноним 27/05/16 Птн 17:54:23  407720
>>407691
Меня вообще смущает эта задача. По идее, в программе для поступления никаких гармонических функций нет. Тащить теперь на экзамен ещё и книжку по урматфизу чтоли?
Аноним 27/05/16 Птн 18:01:46  407727
>>407720
Можно напрямую доказать, что производная функции обращается в ноль, но все равно без комплексной переменной не обойтись. только это скорее теория функции комплексной переменной, а не урматфиз
Аноним 27/05/16 Птн 18:08:22  407737
>>407727
Да, но и этого в программе нет. Там комплексные числа на уровне "Геометрическое изображение, алгебраическая и тригонометрическая форма записи, извлечение корней, корни из единицы". А тут комплексный анализ, который на экзамене можно и не вспомнить, если соответствующую книжку не прихватить. Подстава.
Аноним 27/05/16 Птн 18:40:22  407808
>>407691
Но мы ведь не знаем, верно ли это для всех шаров. В условии сказано, что это выполняется только на окружностях радиуса 1
Аноним 27/05/16 Птн 19:11:13  407872
>>407701

у вас неверное предположение: там не 1 случайная величина, а две: отклонение и случайная точка выстрела

нужно ознакомиться с суммой случайных величин http://www.nsu.ru/mmf/tvims/chernova/tv/lec/node39.html
Аноним 27/05/16 Птн 19:16:36  407893
>>403292
она же вроде простая.
Функция гладкая и ограниченная => есть как минимум один глоб. максимум. Расположим центр окр-ти радиуса 1 в нем. Тогда все точки на окружности равны значению в максимуме.

Также, все точки внутри окружности равны значению в максимуме, т.к. для любой точки внутри окр-ти можно нарисовать окр-ть с центром на той первой окр-ти. Значения НА и ВНУТРИ этой второй окр-ти тоже равны значению в максимуме.

Продолжаем процесс до бесконечности => вся функция равна значению в максимуме.
Аноним 27/05/16 Птн 19:18:17  407902
>>407808
там же написано, в НЕКОТОРОМ шаре
Аноним 27/05/16 Птн 19:24:30  407927
>>407567
>>407567
ты хуесос, ответ 0.96, вскрывайся
Аноним 27/05/16 Птн 19:26:58  407933
>>407727
Там спокойно безо всякого комплана все получается. Элементарным матаном и СМЕКАЛОЧКОЙ
Аноним 27/05/16 Птн 19:49:50  407984
Во сколько завтра начало экза?
Аноним 27/05/16 Птн 20:23:36  408107
>>407984
в 10
Аноним 27/05/16 Птн 20:47:28  408213
наТимураФрунзе, аноны?

Аноним 27/05/16 Птн 20:51:21  408223
5 вариант кто писал? Сколько там порог был, чтоб на очный тур позвали?
Аноним 27/05/16 Птн 22:03:19  408524
14643757998190.jpg (188Кб, 1600x1200)
>>407567
>>407567
ответ 0,936. вот решение
Аноним 27/05/16 Птн 22:40:37  408684
>>408524
Щта ита
Аноним 27/05/16 Птн 23:05:59  408716
>>407872
Как же ты прав! Почему ты не вёл у нас теорвер, ПАЧЕМУ?? Наш старый хуесос такой годноты не объяснял.
Аноним 27/05/16 Птн 23:08:44  408718
>>408524
>>408684
я аналогично решал
Аноним 27/05/16 Птн 23:25:58  408737
>>407872
То что точка выстрела случайна не имеет значение в данной задачи, потому что в условии сказано, что стрелок ВСЕГДА целится в мишень
Аноним 28/05/16 Суб 00:19:31  408769
>>407691
Насколько я помню из урматов, по теореме о среднем из гармоничности следует равенство среднему значению, но не наоборот. Ты не мог бы указать источник обратного утверждения?
Аноним 28/05/16 Суб 00:35:28  408781
>>408769
Ну вообще я из Владимирова смотрел, но на вики тоже есть
https://ru.m.wikipedia.org/wiki/Гармоническая_функция
Аноним 28/05/16 Суб 00:44:55  408785
>>407893
Откуда взялось утверждение про глобальный максимум? Вот у функции распределения нормальной величины нету глобального максимума, она стремится к 0 с одной стороны и к 1 с другой, хотя тоже является гладкой и непрерывной
Аноним 28/05/16 Суб 01:14:50  408802
>>408785
у нее глоб макс бесконечно удаленной точке, очевидно. я, правда, этот случай не рассматривал
Аноним 28/05/16 Суб 05:58:59  408833
>>408737
Ну если стрелок попал не в окаемочку мишени шириной 0.1, то ему пофигу на отклонение. А если попал в окаемочку, то уже не пофигу.
Аноним 28/05/16 Суб 06:45:16  408839
>>408785
Да, тогда непонятно, как ее решать без использования гармонических функций
Аноним 28/05/16 Суб 06:56:34  408841
>>408781
Во владимирове оолько в одну сторону
Аноним 28/05/16 Суб 07:18:23  408842
>>389053
Можно еще определитель расписать по линейности строк. Тогда это будет сумма определителей матриц, в которой строка - это либо строка произведения, либо строка единичной матрицы. каждую такую матрицу можно представить в виде произведения матриц, где в строках и столбцах, в кот. в произведении получается строка единичной матрицы, вектора, ортогональные остальным (такие можно найти, т.к. векторов меньше чем n). дальше Используем, что определитель произведения равен произведению определителей, и сворачиваем все так же, но же другим порядком множителей.
Аноним 28/05/16 Суб 14:45:53  409765
>>408833
>>408833
ну да. поэтому случайное остается только отклонение. И надо расссматривать эти области мишени отдельно
Аноним 28/05/16 Суб 14:57:29  409831
>>408841
Там дальше есть в обратную сторону.
Владимиров, Жирнов, УрМатФиз, параграф 8 Дальнейшие свойства гармонических функций, пункт б
Шад Аноним 28/05/16 Суб 15:40:57  409959
а есть у кого-нибудь вариант от 28 мая? скиньте пж
Аноним 28/05/16 Суб 15:44:23  409966
Скажите, а если объебался в задаче чисто из-за невнимательности и из-за этого ответ неверный, как они это оценивают?
Аноним 28/05/16 Суб 16:04:01  409996
>>409831
Жаринов*
Спасибо, брат, но это следствие не подходит, т.к. формулировке указано: существует r_0, такой что для ЛЮБЫХ радиусов r<r_0 верно осреднение, в условии же задачи только для одного радиуса это выполняется
Аноним 28/05/16 Суб 16:05:41  410001
14644407419000.jpg (2262Кб, 3264x2448)
Вариант за сегодня.
Аноним # OP  28/05/16 Суб 16:22:57  410027
Я хуй и мудак
Аноним 28/05/16 Суб 16:32:53  410050
>>410027
почему?
Аноним 28/05/16 Суб 16:33:32  410052
пацаны, как задачу с усреднением по окружности-то решать ?
Аноним 28/05/16 Суб 17:27:49  410181
>>404911
Объясните, пожалуйста, как здесь применяется линейное свойство оприделителя
Аноним 28/05/16 Суб 17:57:56  410299
>>410001
Спасибо, что выложил, брат. 1, 2, 3 изи. 5ую через монотонность доказывал несуществование? 8ая пиздец жесть какая-то
Аноним 28/05/16 Суб 20:20:15  410830
Аноны, как решается 2-ая задача из варианта 21 мая? Наверное я тупой.
Аноним 28/05/16 Суб 20:20:58  410833
>>410001
Как доказать 1-ую задачу для вырожденных A и B?
Аноним 28/05/16 Суб 20:26:23  410850
>>410833
Нет, я неправильно задал вопрос, там скорее всего ничего доказывать не нужно, а нужно проявить СМЕКАЛОЧКУ и найти такие вырожденные матрицы 2х2 или 3х3, для которых импликация неверна. Подскажите, люди добрые, эти "ваще изи, сразу написал как увидел" матрицы плс
Аноним # OP  28/05/16 Суб 20:34:59  410881
>>410050
Тому що почти ниче не решил
Аноним 28/05/16 Суб 20:49:59  410927
>>410850
A = [[1,0],[0,0]]; B - единичная матрица => A не равно B
Аноним 28/05/16 Суб 20:55:07  410946
>>410927
Да, это оно! Спасибо!! Я перебрал все матричные единицы, а вот про единичную матрицу не подумал...
Аноним 28/05/16 Суб 21:08:20  410968
>>410850
чет я не понял твою идею(((
ну найду я для вырожденных матриц такую которая будет не удовлетворять. и при импликации при истинном условии и ложном выводе мы получаем ложь, а что это дает?
Аноним 28/05/16 Суб 21:10:12  410970
>>410968
пурвое условеи будет удовлетворять, а второе нет
Аноним 28/05/16 Суб 21:31:20  411006
>>410970
аааа блин, точно, т.е. ответ неверно
Аноним 28/05/16 Суб 21:48:53  411067
>>409996
>>410001
кто решил задачу на сходимость? В чем основная идея? Не могу придумать, что с этими логарифмами делать
Аноним 28/05/16 Суб 21:50:17  411073
>>411067
там надо над экспонентой все вытащить и ряд упростится очень сильно
Аноним 28/05/16 Суб 21:52:57  411078


четвертая задача вроде тоже не сложная, хотя может ошибаюсь.

создадим два массива длины n
в первый положим индексы правых концов, отсортированные по возрастанию, во второй индексы левых концов, отсортированных по убыванию

далее идем начиная с 1000 элемента по обоим массивам.
как только в просмотренных элементах оказалась 1000 одинаковых индексов в обоих массивах, то проверяем есть ли ещё индексы, не участвовавшие до этого , если есть, то элемент с нужной вложенностью присутствует, иначе отсутствует
Аноним 28/05/16 Суб 21:56:00  411080
>>411078
Точно также решал
Аноним 28/05/16 Суб 22:29:32  411171
>>411073
Я можно поподробнее? Я додумался только как от степени избавиться с помощью экспоненты. Но вложенные логарифмы все равно остаются
Аноним 28/05/16 Суб 22:36:36  411206
>>411171
каждый член можно переписать так

(ln ln n)^-(ln n)=e^((-ln n)*(ln ln ln n))=
n^(-ln ln ln n)

начиная с какого то N степень будет меньше например -2 , а значит оставшаяся часть ряда будет меньше сходящегося ряда. Значит и этот ряд сходится
Аноним 28/05/16 Суб 22:53:02  411256
>>411206
Спасибо тебе, дружище!
Аноним 28/05/16 Суб 23:33:21  411354
>>410299
Да, в 5ой три варианта рассмотрел -
1. f монотонна
2. существуют x1 и x2 такие что f(x1) =f(x2)
3. существуют такие x1, x2, x0, что f(x1) <f(x0)>f(x2) или f(x2) >f(x0)<f(x1), x1 < x0 < x2
Разбираешь и все получается.
8ая как раз простая, если я нигде не налажал. рассматриваем g(t) = f(t)(x0, x0), где f - кривая, соединяющая A и B. Расписываешь, и получается, что g(t) - непрерывна по t. g(0) = A(x0, x0) >0, g(1) = B(x0, x0) < 0, g непрерывна, значит есть t0: g(t0)=0 значит f(t0)(x0, x0)=0 значит f(t0) вырожденная.
Аноним 28/05/16 Суб 23:42:02  411369
>>410299
На этих изи тоже ска объебаться можно оказывается.Я блять как третью увидел, сразу начал хуярить плотности X+Y, X-Y, нахуярил, поошибавшись, а потом смотрю и думаю, а нахуя они мне. И по-быстрому решил графически. Ну сука ну нахуя было кидаться, не подумав. Час на этой хуйне наверно проебал(( Ска дебил блять аж бесит нахуй
Аноним 28/05/16 Суб 23:47:46  411381
>>411354
ну это я примерно так себе и представлял, но как определить непрерывность для функционалов ?
Аноним 28/05/16 Суб 23:54:10  411396
>>411369
>И по-быстрому решил графически
можешь пояснить как? я часа полтора убил пока свертки считал
Аноним 28/05/16 Суб 23:56:14  411402
>>411381
Там любую норму можно было брать, какую захочешь
Аноним 29/05/16 Вск 21:41:09  414997
кто решил 8 задач на изичах за 20 минут и еще задач попросил у оргов?
Аноним 29/05/16 Вск 22:10:29  415106
>>414997
а че были такие?
Аноним 29/05/16 Вск 23:14:20  415324
>>410001
По условия 6 задачи, я правильно понимаю, что крайние предметы, которые выбрали, а следующий за ним нет, то он считается за хороший? То есть. Пусть 1 - предмет выбрали, 0 нет. В последовательности (10101) Х=3, а не Х=1?
Аноним 30/05/16 Пнд 02:12:21  415827
>>407002
Делишь мишень на 9 кусков (4 угла, 4 края, 4 центра), и в каждом считаешь.
Аноним 30/05/16 Пнд 11:02:16  416503
>>403217
>>404908
Аноним 30/05/16 Пнд 13:17:12  416805
>>408524

Вопрос по поводу расчёта P1. При x=0.1 подинтергральное выражение принимает значение 0. То есть получается, что целясь в верхнюю границу S1, мы попадем в мишень с вероятностью 0. Но это же неправда, т.к. по идее вероятность там 1/2
Аноним 30/05/16 Пнд 14:56:55  417280
>>415106
Были те, кто за 2.5 часа все решил
Аноним 30/05/16 Пнд 15:24:54  417412
>>417280
а много народу было 28?
Аноним 30/05/16 Пнд 17:01:38  417969
>>411354
Из того, что значение формы в некоторой точке ноль, не следует, что ее матрица вырожденная
Аноним 30/05/16 Пнд 19:10:51  418558
>>417280
А ты уверен, что они ВСЕ решили, а не просто забили и ушли?
Аноним 30/05/16 Пнд 20:32:47  418968
>>417969
Если это происходит для не нулевого вектора, следует. Потому как иначе форма либо положительно, либо отрицательно определена по соответственно положительному и отрицательному критериям сильвестра
Аноним 30/05/16 Пнд 20:39:36  419011
>>418968
X1^2 - X2^2 положительно или отрицательно определена?
Аноним 30/05/16 Пнд 20:51:54  419084
>>419011
Извиняюсь.
Аноним 30/05/16 Пнд 20:59:38  419128
>>410001
Вопрос дня: как решать 6-ую?
Аноним 30/05/16 Пнд 21:47:58  419320
>>417969
мда точно лоханулся так лоханулся тут. Надо придумать тут короче какую-то функцию, которая для положительно определенной формы положительная, для отрицательной - отрицательная и нулевая - для вырожденной.
Аноним 30/05/16 Пнд 22:19:16  419398
>>419320
Бля сукпздц, можно же взять например минимальное по модулю собственное значение, - это сшитые две непрерывные функции коэффициентов билинейной формы, и в точках сшития эта функция непрерывна, значит, она и вся непрерывна. Отрицательна для отрицательно определенной, положительна для положительно определенной и ноль для вырожденной. И по предыдущим рассуждениям дальше. Ска ебаные 4 часа на решение. Скапздц у меня бомбит.
Аноним 30/05/16 Пнд 22:31:42  419420
какая у x степень в 5 задании? 1 - гамма? не могу разглядеть
Аноним 30/05/16 Пнд 22:53:00  419447
>>419420
1-x^3
Аноним 30/05/16 Пнд 23:27:49  419561
>>419084
прощен )
Аноним 30/05/16 Пнд 23:33:45  419575
>>410001
алло ботаны, как решать 6 и 7?
Аноним 30/05/16 Пнд 23:46:37  419588
>>419398
блять, спасибо, чувак, походу твое решение правильное, по карйней мере не видно недочетов так сразу, а я уже сука второй день думал над этой задачей
Аноним 31/05/16 Втр 16:14:13  420233
>>419084
>>419575
Если что 7 это задачка для 8 класса. Решается элементарной логикой
Аноним 31/05/16 Втр 19:24:09  420551
>>420233
С каких пор в 8 классе проходят графы?
Аноним 31/05/16 Втр 20:13:05  420689
Аноны, какой ответ в 6 задаче из варианта 28? У меня (n-k+2)k(n-k)/(n(n-1))
но я в нем сильно сомневаюсь
Аноним 31/05/16 Втр 21:12:21  420783
>>420551
Ну так она не на графы, а на СМЕКАЛОЧКУ.
В общем в каждой вершине идешь по часовой стрелке и красишь дороги в цвета в определенном порядке(красный, синий, зеленый например). Видно, что инквизитор на дороги какого-то цвета заезжать не будет, их можно вообще убрать. А граф, в котором у каждой вершины 2 ребра это цикл.
Аноним 31/05/16 Втр 21:26:24  420796
>>420783
>>Видно, что инквизитор на дороги какого-то цвета заезжать не будет.
Это ещё почему?
Аноним 31/05/16 Втр 21:30:51  420803
>>420796
забей, он хуйню вещает
Аноним 31/05/16 Втр 21:36:12  420809
>>420796
>>420783
чувак шарит ёпт,
берешь тройку (R,G,B) - цвета дорог по часовой стрекле в городе Э, идёшь по дороге R, красишь там две оставшиеся дороги, потом два варианта G или B, пусть G, то есть первые две дороги R->G, но в этой тройке (R,G,B) дорога G - правая для R, а дорога R - левая для G, имеем цепь R->G->R->G->R->... Если в какой-то момент мы попали на дорогу B, значит возможны два варианта R->G->B или G->R->B, первый хуйня потому что два раза вправо повернул, второй хуйня потому что два раза влево повернул
Аноним 31/05/16 Втр 21:46:57  420825
>>420809
Нарисуй граф из четырех вершин и любой путь.
Аноним 31/05/16 Втр 21:57:41  420848
>>420825
да, хуйня...
Аноним 31/05/16 Втр 21:58:02  420849
>>420809
господи иисусе... ты когда дороги красишь докажи , что дорога, покрашенная в В при первом на нее заходе, не совпадет с R или G для другой вершины. как я и говорил >>420803
Аноним 31/05/16 Втр 22:01:01  420857
>>420849
ну лан, ебанул не подумав, не сердись
Аноним 31/05/16 Втр 22:04:14  420861
Я бы доказывал от противного: рассмотрел бы любой путь братана и любой цикл в нем, если цикл содержит четное число вершин, то братан выйдет из него с первого раза, если нечетное, то со второго раза. Ну а дальше сами)
Аноним 31/05/16 Втр 22:15:30  420876
>>411078
Как искать эти одинаковые индексы?
Аноним 31/05/16 Втр 22:18:25  420878
>>415324
Да, верно.
Аноним 31/05/16 Втр 22:24:10  420886
сука, пиздец я конч, осталось три дня я еще не начинал готовиться серьезно. пацаны, посоветуйте какие хоть книжки в библиотеке взять ?
Аноним 31/05/16 Втр 23:33:46  420959
>>420876
просто на каждом шаге находим например пересечение множеств индексов из начала первого списка и начала второго списка. и считаем его мощность
Аноним 31/05/16 Втр 23:43:57  420962
>>420959
И чему равна сложность нахождения пересечения множеств индексов?
Аноним 01/06/16 Срд 00:12:22  420983
>>420876
можно отсортировать a и b (время позволяет) и после этого даже тупой перебор будет проходным по времени. а задачи сделать "оптимально" - нет
Аноним 01/06/16 Срд 00:55:25  421006
>>420983
Ты что перебирать собрался?
Аноним 01/06/16 Срд 02:38:09  421029
>>420876
бинарным поиском епт. массивы то упорядочены
Аноним 01/06/16 Срд 07:29:51  421054
>>420962
не поверишь, o(n ln n)
например тупо сложил оба набора в один массив , отсортировал а потом прошелся разок и выкинул дубли.
Аноним 01/06/16 Срд 07:33:01  421055
>>421054
я немного натупил.
держишь всегда упорядоченный массив, и при добавлении за ln n находишь место вставки и вставляешь элемент в множество, если его ещё там не было. в сумме получишь сложность не более n ln n
Аноним 01/06/16 Срд 20:05:02  422163
>>420689
(n-k)k(n-k+1)/(n(n-1))
Аноним 01/06/16 Срд 21:51:37  422398
>>403165
Анончики, дорогие, какой же ответ ожидается в первом задании? x = 0? x(n) = o(1/n)? x(n) ~ (2pik/n)?
Аноним 01/06/16 Срд 22:08:30  422426
ребят, скажите, пожалуйста, как третью решать без нахождения распределений Х+Y, Х-Y?
Аноним 01/06/16 Срд 22:09:55  422430
>>422398
ты не можешь фиксированный х выражать через n, которое меняется. я бы ответил 2pik, k in Z
Аноним 01/06/16 Срд 22:10:43  422431
сходил сегодня книжек набрал, библиотекарша вошла в ступор, спросила у меня гос что ли)
Аноним 01/06/16 Срд 22:18:36  422442
>>422430
Но при x = 2pik и n → ∞, nx не будет стремиться к 2pik, единственное подходящее фиксированное значение для x это 0
Аноним 01/06/16 Срд 22:44:44  422490
>>422442
а почему он должен стремиться к 2pik?
Аноним 01/06/16 Срд 23:04:44  422526
>>422490
потому что lim(cos(nx)) = 1 => lim(nx) = 2pik
ЧЯДНТ?
Аноним 01/06/16 Срд 23:29:07  422557
>>422442
чувак, вот реально, либо ты троллишь, либо если уж ты собрался поступать в шад надо бы знать, что такое периодическая функция
Аноним 01/06/16 Срд 23:29:32  422558
>>422426
>>422426
>
ну плииииз
Аноним 02/06/16 Чтв 00:06:56  422589
>>422557
2pik было бы ответом, если бы n принимало только целые значения, а при действительных n предел не сходится, разве нет? эта ваша периодическая функция так и будет скакать от -1 до 1 на бесконечности
Аноним 02/06/16 Чтв 00:09:16  422593
>>422589
просто иди нахуй мамкин тролль
Аноним 02/06/16 Чтв 00:30:10  422601
>>422593
да хуй с ним с вашим шадом, можете ссать мне на лицо, кормить говном, только объясните эту хуйню
Аноним 02/06/16 Чтв 03:02:30  422632
>>422601
Челик, ты серьезно? Никто не будет буквой n обозначать действительное число. n натуральное понятное дело
Аноним 02/06/16 Чтв 10:09:55  422744
>>411369
брат, в третьей Х+У и Х-У независимые случ.величины ведь ?
Аноним 02/06/16 Чтв 13:37:44  423059
>>422163
брат, как получить-то ? что-нибудь наводящее, плз
Аноним 02/06/16 Чтв 13:38:14  423062
>>422601
это предел последовательности а не функции, чувак
Аноним 02/06/16 Чтв 13:38:45  423065
>>422558
забейте, я посчитал и так вроде не сложно
Аноним 02/06/16 Чтв 16:37:12  423408
>>423059

http://mathkonspekt.blogspot.ch/2014/02/blog-post_28.html

очень похожая задача тут
Аноним 02/06/16 Чтв 17:07:13  423448
>>423408
господи, нихуя не понял в чем связь, но хорошо что там есть разбор задач шад
Аноним 02/06/16 Чтв 18:43:40  423639
>>390045
У нас тут чё, всерос какой-то? Ну ёп твою мать! Как же задолбало матшколиё. В жизни оно так не взлетает.

Вот тебе задача из жизни: есть двумерная сфера, в ней k проколов, туда я вклеиваю листы Мёбиуса, по границе листа клею. Получается, внезапно, гладкое многоборазие. Нужно вычислить его первые и вторые когомологии де Рама.

Жду красивое решение.
Аноним 02/06/16 Чтв 20:13:45  423840
в
Аноним 02/06/16 Чтв 22:18:36  424031
>>423408
объясни плз в чем связь между этими задачами.
Ты как-то решал через линейность мат ожидания?
Аноним 02/06/16 Чтв 22:32:30  424048
>>424031
да, я просто написал, что число таких элементов без соседей равно сумме по событиям, что i-ый элемент без соседей. потом взял мат. ожидание и все. сорри, что как таджик пишу, но я и есть таджик
Аноним 02/06/16 Чтв 23:14:54  424084
>>423639
внезапно на семинарах по дифгему с носовским разбирали
Аноним 02/06/16 Чтв 23:23:55  424086
Господа, а какой средний курс людей, обучающихся на ШАДе? Скажем, много людей, параллельно магистратуре изучающих?
Аноним 02/06/16 Чтв 23:39:46  424102
>>424086
Большинство физтехов поступают туда на 2-3 курсе.
Аноним 02/06/16 Чтв 23:41:59  424105
>>424102
Да там что физтех, что нет, любой технический вуз первые два курса это вся программа поступления. Только времени нет, а в магистратуре — есть. Вот и вопрос.
Аноним 03/06/16 Птн 00:40:45  424127
>>424048
Для любого элемента кроме крайних вероятность оказаться без соседе это 1- вероятность наличия в выбранном множестве элемента и его соседа справа - вер-сть наличия элемента и его соседа слева + вер-сть наличия соседеней справа и слева . Первые две вероятности это С из n-2 по k-2 / C(n,k), последняя вероятность это C(n-3, k-3)/C(n, k). Для крайних элеменов соответственно может быть сосед только с одной стороны. Суммирую мат. Ожидания по всем элементам, получаю хуйню, хотя знаменатель тоже n(n-1). Анон, ЧЯДНТ?
Аноним 03/06/16 Птн 00:49:17  424128
>>424084
Наверняка никаких выебонов, а стандартной техникой. Майера — Вьеториса?
Аноним 03/06/16 Птн 01:35:01  424134
>>424102
Сдается мне, что далеко не большинство
Аноним 03/06/16 Птн 01:35:21  424135
>>424086
Большинство
Аноним 03/06/16 Птн 01:36:52  424136
>>424102
да чё уж там, все нахуй, всем физтехом
Аноним 03/06/16 Птн 09:23:25  424167
>>424048
спасибо! все понял!
Аноним 03/06/16 Птн 09:57:56  424175
>>424167
>>424127
у тебя получилось ? ты делал так же?
Аноним 03/06/16 Птн 10:40:15  424197
>>424175
все, я сам решил
Аноним 03/06/16 Птн 10:40:47  424199
>>424134
>>424136
я на 5 пытаюсь поступить
Аноним 03/06/16 Птн 11:12:15  424240
>>424167
>>424127
>>424175
>>424197
Ребят, подскажите всё-таки правильный способ расчёта вероятности в 6ой
Аноним 03/06/16 Птн 12:47:24  424360
>>424240
Вводишь функцию Ij от исхода, которая равна единице, если j элемент выбран, а соседние нет. Ее матожидание найти легко, а сумма матожиданий таких функций равна тому матожиданию, которое мы ищем.
Аноним 03/06/16 Птн 13:21:02  424389
>>424240
короче, 1) искомая случайная величина это сумма случайных величин единица если i эл-т включен без своих соседей, 0 в других случаях, тогда мат. ожидание есть сумма всех таких мат ожиданий от 1 до n.

2) допустим мы выбрали какой-то элемент, вероятность быть выбранным без соседей для него, если имеется два соседних элемента, это 1 - p(сосед слева включен вместе с этим элементом) - p(сосед справа...) + p( оба соседа), если сосед один(элемент с краю нашего множества) соответственно слагаемых меньше. еще нужно домножить на вероятность при случайном выборе захватить сам рассматриваемый i-й элемент

суммируешь, все красиво сокращается
это мой предыдущий пост, там цешки выписаны правильно, лень заново писать>>424127
Аноним 03/06/16 Птн 14:16:15  424424
аноны, блять, господи, как решать задачу про функцию равную своему среднему по окружности, скажите плз, трубы горят
Аноним 03/06/16 Птн 14:22:10  424430
>>424424
Ты тред читать пробовал? Выше писали, что из теоремы Лиувилля и свойства среднего гармонической функции очевидно следует утверждение этой задачи.
Аноним 03/06/16 Птн 14:47:19  424476
>>424360
>>424389
Спасибо!
Только вопрос - почему в формуле для вероятности мы начинаем расчёт с 1? Разве не с вероятности элемента ai быть выбранным? (С из n-1 по k-1)
Аноним 03/06/16 Птн 14:49:25  424479
14649545658150.png (103Кб, 2743x1018)
>>424430

чувак, я-то тред читал от начала и до конца. кто-то приводил свойство гарм. функций из Владимирова, но деле в том, что мы не можем доказать гармоничность по этому свойству, оно не соответствует нашей ситуации. если же я неправ, то скажи в чем, плз. вырезал специально из учебника свойство на которое ссылался анон. в нашем случае усреднение верно только по одной окружности, а надо же по любой окружности с радиусом меньшим r_0
Аноним 03/06/16 Птн 14:50:59  424482
>>424476
не понял вопроса. мы в сумме перебираем слeчайные величины которые соответствуют предметам под номерами от 1 до n
Аноним 03/06/16 Птн 15:03:45  424503
>>424479
А, точно. Хуй знает тогда, как это решать..
Аноним 03/06/16 Птн 15:04:39  424505
Аноны, Кто-нибудь решал задачу 3 из варианта 28 мая? Какой ответ получился? У меня 7/9
Аноним 03/06/16 Птн 15:10:30  424508
>>424505
я ответ не досчитал
Аноним 03/06/16 Птн 15:18:10  424517
>>424505
подсчитал только что, короче получилась какая-то лютая хуета 312835/(36^3). ты через распределения суммы и разности считал ?
Аноним 03/06/16 Птн 15:18:56  424520
>>424517
тут знаки умножения, просо то что между звездочками превратилось в курсив
Аноним 03/06/16 Птн 15:35:10  424540
>>424482
нет, я про выражение "1 - p(сосед слева включен вместе с этим элементом) - p(сосед справа...) + p( оба соседа)". По-моему на первом месте должно быть p(элемент включен), и ни на что больше домножать не нужно
Аноним 03/06/16 Птн 15:55:35  424577
>>424540
ааа, так я про это и говорил. нужно все это вместе домножить на c(n-1,k-1)/c(n,k), ты прав короче. просто, я не совсем точно описал, здесь p - вероятности выбора соседа, при условии того что сам элемент уже выбран
Аноним 03/06/16 Птн 18:56:03  424849
блееееа, я просто физически не могу начать двигаться дальше пока не решу задачу про усреднение по окружности сук...
Аноним 03/06/16 Птн 19:38:49  424931
>>424849
я забил на нее и тебе советую
Аноним 03/06/16 Птн 20:41:02  425144
Господа, а зачем вы хотите в ШАД? Мне интересна ваша мотивация: попасть в "тусовку", работать в Яндексе, или что-то еще? Просто объем усилий для поступления крайне немаленький, год-два быстро пролетающей молодости, а профит не очевиден (сугубо личное имхо)
Аноним 03/06/16 Птн 21:40:00  425319
>>425144
я учусь на физика, но не хочу работать по специальности всю жизнь за 30к в ультрабюрократической конторе, при этом получив секретность выше 3 формы допуска, нахуй надо, лучше поработаю быдлокодером в офисе. я должен до выпуска (остался 1 год) устроиться на работу девелопером, если с шад не получится, буду пробовать другими путями, но шад имхо самый простой вариант вкатиться, тут тебе и программа обучения и домашки и стажировка и т.п.
Аноним 03/06/16 Птн 21:42:13  425326
>>425144
>>425319
и да, я итак проебал 6 лет своей молодости на игруньки в доту и всяческие занятия спортом/игры, по крайней мере касательно шада у меня конкретная цель, не думаю что это трата времени
Аноним 03/06/16 Птн 21:42:53  425329
>>425144
>>425326
и я все еще не решил задачу с осреднением по окружности сука ))
Аноним 03/06/16 Птн 21:44:53  425334
28 кстати, по-моему легче вариант, чем был 21, может завтра еще легче будет ))
Аноним 03/06/16 Птн 22:25:13  425425
>>425334
Нихуя, у них многолетняя традиция во второй день давать самый простой вариант=)
Аноним 03/06/16 Птн 23:07:13  425484
Короче, аноны, удачи кто завтра пишет! Пусть победят самые потные из потных!
Аноним 03/06/16 Птн 23:46:49  425526
>>425425
так а хули вы молчали сукиииии
Аноним 03/06/16 Птн 23:47:13  425527
>>425484
сука в 7 утра вставать чтобы из своей жопы доехать туда
Аноним 04/06/16 Суб 00:14:55  425552
>>425526
Ну так проходной балл для каждого дня свой, так что, какая в общем-то в жопу разница.
Аноним 04/06/16 Суб 11:16:10  425756
>>425552
Т.е. набирают лучшие эн процентов с каждого дня что ли? А сколько человек обычно набирают? Там за каждую задачу разное количество баллов?
Аноним 04/06/16 Суб 12:17:45  425928
больше экзаменов не будет? 4 июня -- последняя дата?
Аноним 04/06/16 Суб 14:19:45  426202
>>425928
5 июня еще заочный же еще
Аноним 04/06/16 Суб 15:32:40  426315
Вбросьте вариантик
Аноним 04/06/16 Суб 17:13:53  426506
сука блядь не поступлю я в шад в этом году. аноны накидайте ответов, плз, надеюсь хоть 3 задачи у меня правильные
Аноним 04/06/16 Суб 17:18:10  426519
>>426506
я короче как даун пытался полтора часа подсчитать задачу с тремя случайными величинами, хотя надо было другие решать... въебите мне плз
Аноним 04/06/16 Суб 18:21:29  426709
Я вроде 5 решил. Про 3 случае. Величины решал геометрически
Аноним 04/06/16 Суб 18:31:01  426722
>>426709
ну напиши ответы, брат. я сравню хоть
Аноним 04/06/16 Суб 18:31:54  426725
>>426709
как ты кстати геометрически решал ?
Аноним 04/06/16 Суб 19:01:31  426807
14650560913630.jpg (785Кб, 2039x2905)
Аноним 04/06/16 Суб 19:54:40  426961
боже, какой я тупой. Как учить статистику и прочее вероятностное говно? Больше, чем на шары в ящике, я не понимаю, ковариации, хуеряции, фу...
Аноним 04/06/16 Суб 20:05:09  427009
>>426961
ботаны, ну плз, напишите ответы к новому варианту
Аноним 04/06/16 Суб 20:05:27  427010
тем кто 28 и 21 писал уже пришли результаты ?
Аноним 04/06/16 Суб 21:30:38  427426
14650650385830.png (19Кб, 713x204)
>>426807
Ну что, господа, у кого хватило смекалочки?
Аноним 04/06/16 Суб 21:58:05  427552
>>427426
я просто нарисовал эту функцию, т.е показал принцип её построения
Аноним 04/06/16 Суб 22:00:49  427565
>>426722
собственно выше тож мой коммент

первая легко решается, возьмем число k отличное от диагональных элементов и 0

первая матрица это все элементы под диагональю и k на диагонали

вторая матрица все элдементы над диагональю и диагональные минус k на диагонали

т.е. обе матрицы не вырождены а значит обратимы
Аноним 04/06/16 Суб 22:06:00  427598
>>426725
там надо рисовать куб соответсвенно вероятность равна объему части куба

пришлось рассматривать несколько случаев
t<=0 тогда p=0
0<t<=1 что-то в духе t^2/6
1<t<=2 t^2/6 - что-то от t
и последний случай t>2 там еще сложнее формула щас не воспроизведу

смысл такой рещается графически на тех же принципах , что тут,http://sernam.ru/book_tp.php?id=60 только в пространстве
Аноним 04/06/16 Суб 22:09:00  427624
>>426722
про ковариации, по определению все расписывается, а дальше я делал такой финт. типа данное выражение от самих значений зависеть не должно, т.к. они не влияют на завсимость случ величин. поэтому взяв в качестве значений разные вариации 0 и 1 получим, что для всех 4 вариантов p(xy)=p(x)p(y)
Аноним 04/06/16 Суб 22:33:23  427770
>>427426
а если я доказал несуществование это хуево ? )
Аноним 04/06/16 Суб 22:36:18  427783
>>427770
ну для двух то не существует, а для трех существует.
Аноним 04/06/16 Суб 22:37:40  427792
>>427783
а если я доказал для обоих случаев? тут либо я даун, либо ты мне втираешь какую-то с функциями, которые не являются непрерывными
Аноним 04/06/16 Суб 22:40:17  427807
>>427792
втираешь какую-то дичь * фикс
Аноним 04/06/16 Суб 22:41:18  427815
хоть я и соснул на экзамене в шад, зато поход на варкрафт меня порадовал, закон сохранения радости блеать
Аноним 04/06/16 Суб 22:44:16  427827
>>427807
тот пример выложил не я, но функция такая существует

возьми волновую фнкцию и пусть она имеет возрастающий тренд. При этом каждый следующий локальный минимум равен предпоследнему локальному максимому.
тогда как раз получишь фкнцию котрая принимает каждое значение ровно 3 раза
Аноним 04/06/16 Суб 22:47:02  427843
14650696228510.png (10Кб, 1152x648)
>>427792
даже зарисовал схематично
Аноним 04/06/16 Суб 22:50:59  427864
>>427843
пиздец я эпично соснул... такие рисунки фигурировали в моей доказательстве отсутсвия таокй функции (((
Аноним 04/06/16 Суб 22:51:15  427866
>>427864
моем*
Аноним 04/06/16 Суб 22:54:00  427885
>>427843
в общем, пиздец, теперь точно не поступлю...
в 6 случаем сигнатура не (+,-,0,0,0,...)? а как предел в 7 получить ? ответ я уже узнал, но как получить ?
Аноним 04/06/16 Суб 22:57:07  427909
>>427885
я к сожалению решил только первые 5 задач, остальные, не решил, да и времени на них уже особо не хватило. Какой кстати ответ в пределе. я предел пытался решить делением на n интегралов, т.к. подинтегральная функция разрывна, а дальше видимо много раз по частям.
Аноним 04/06/16 Суб 23:00:47  427948
>>427885
в 6ом правильно.
в 7ом надо показать, что при больших n можно считать степенную часть постоянной на участках, где аргумент экспоненты меняется на единицу. тогда экспоненту можно заменить просто на число равное усредненному значению экспоненты
Аноним 04/06/16 Суб 23:01:49  427954
>>427909
ответ (e-1)/2017.
Аноним 04/06/16 Суб 23:04:20  427974
>>427885
объясните плз 6ое
Аноним 04/06/16 Суб 23:09:56  428005
>>427948
>>427909
только я 6 решал в последние полчаса и поэтому вместо n+1 элементов вектора написал n. седьмую задачу тоже под конец пытался решить, мои аргументы были:
1)забьем на точки разрыва, их счетное множество, интеграл не поменяется если мы их не учтем
2)на каждом интервале (k/n, (k+1)/n) k < n -1 экспонента дифференцируемая
3) зафиксируем n_0, проинтегрируем по частям экспоненту 2017 раз, получим много слагаемых , но у всех n_0 в знаменателе, устремим n_0 к бесконечности, получаем 0.

тут я такой короче выхожу с экзамена и... блять, господи, наша экспонента всегда больше единицы, значит интеграл как минимум больше 1/2017, короче пиздец, мне стыдно перед человеком, который будет проверять мою работу.

если честно, не понял про усреднение. че там пацаны, что с раскрашенным графом? рекурсивный алгоритм? я, даун, блять даже не решал этот номер на экзамене....
Аноним 04/06/16 Суб 23:12:14  428021
>>428005
кстати, в чем я проебался с интегралом? с тем что не учел точки разрыва ?
Аноним 04/06/16 Суб 23:17:48  428058
>>427885
Почему точно не поступишь? Сколько решил?
Аноним 04/06/16 Суб 23:19:03  428065
>>428005
в графе я юзал рекурсивный алгорит.
а втвоем решении с интегралом верхняя граница должна быть не до i/n а до i/n-eps
Аноним 04/06/16 Суб 23:30:01  428129
>>428058
ну я думал я 4 решил, но по итогам лоханулся с пунктом б) про непрерывные функции и 6 не очень решил + оставил там 2, засранных в процессе решения задачи на 3 случ.величины, листа, наврят ли там что-то наберется. короче я оцениваю свои баллы ~3 задачи. если учесть, что проходной вроде как 4, я соснул тунца, но не это обидно, а то что я ведь мог решить остальные задачи, просто зря считал в лоб эту вероятность ебаную, короче печаль. в принципе, я доволен, опыт получил, в следующем году попробуюсь, уже меньше стресса для меня будет, не буду так волноваться как в этом году, лучше покажу себя.
Аноним 04/06/16 Суб 23:30:17  428132
>>428129
я спать нахуй
Аноним 05/06/16 Вск 11:33:34  428770
резалты пришли
Аноним 05/06/16 Вск 11:41:43  428779
>>428770
тем, кто 21 писал?
Аноним 05/06/16 Вск 12:15:39  428836
ну что, кто пишет сейчас онлайн?
Аноним 05/06/16 Вск 12:19:52  428842
>>428836
ага
Аноним 05/06/16 Вск 12:23:29  428848
>>428779
я 28 писал, сегодня утром пришли.
Аноним 05/06/16 Вск 12:24:59  428856
Кто сейчас пишет, может поделитесь ответами?)))
Аноним 05/06/16 Вск 12:55:58  428924
>>428856
охуевшие пидоры
Аноним 05/06/16 Вск 12:56:14  428925
>>428848
>>428770
прошел ?
Аноним 05/06/16 Вск 13:22:43  428990
>>428925
Пойду щемить яндексовских ботанов на аппеляции, должны как минимум одну задачу накинуть, а то охуели
Аноним 05/06/16 Вск 13:25:15  429005
>>428990
так че по результатам ? какой проходной ?
Аноним 05/06/16 Вск 13:36:13  429030
>>429005
Хер знает, какой проходной. Написано после 16 скажут.
Аноним 05/06/16 Вск 14:44:13  429207
по факту получается 21 были самые сложные задачи? а 4 самые легкие
Аноним 05/06/16 Вск 14:54:10  429238
кто-нибудь поделится заочным вариантом?
Аноним 05/06/16 Вск 14:55:38  429247
>>429238
заочным? это тем что онлайн идет сейчас?
Аноним 05/06/16 Вск 15:02:33  429278
Пацаны, кто решал вариант от 28го, посмотрите вторую задачу, про сходимость ряда. Там же ряд сходится, да? Или меня жестко глючит? Просто минус за эту задачу стоит.
Аноним 05/06/16 Вск 15:19:34  429360
>>429247
да
Аноним 05/06/16 Вск 15:24:38  429385
>>429360
так ты в яндекс контекст зайди, и нажми кнопку зарегестрироваться, он для всех открыт
Аноним 05/06/16 Вск 15:25:50  429393
>>429385
https://contest.yandex.ru/contest/2454/enter/
Аноним 05/06/16 Вск 16:04:21  429601
бля, протупил-протормозил, решил половину, хотя можно было и больше

короче, заочный вариант просто неприлично лёгкий, никаких тебе ковариаций-хуяриаций и прочей поебени, которой заполнен тред

думал, вообще ничего не решу, но теперь чувствую себя не таким полным дауном, как на самом деле

дискасс
Аноним 05/06/16 Вск 16:06:44  429618
>>429601
сколько баллов набрал? ответы сразу проверило у меня.
Аноним 05/06/16 Вск 16:08:13  429626
>>429618
бля. только что все вердикты показывал где ОК где WA. обновил страницу, хуяк и везде решение принято на проверку
Аноним 05/06/16 Вск 16:09:10  429631
14651321506010.webm webm file (659Кб, 640x360, 00:00:07)
>>429601
Сколько отправил? Сколько думаешь правильно.
Вообще кто сегодня писал, делимся успехами и впечатлениями.
Аноним 05/06/16 Вск 16:10:18  429640
>>429631
в 7.1 какой верный ответ?? написал 3 - неверно
Аноним 05/06/16 Вск 16:11:11  429645
>>429640
Как ты узнал, что не верно? Там же написано принято на проверку и результата нет.
Аноним 05/06/16 Вск 16:11:13  429646
>>429626
В текстовых показывало ошибку формата тоже? Кроме них и задачи E все правильно. Ебал эту комбинаторику, никак не мог найти правильную формулу для такого типа строк.
Аноним 05/06/16 Вск 16:12:11  429649
>>429640
Сложность NlogN, сколько там правильно сам считай.
Аноним 05/06/16 Вск 16:12:48  429654
>>429646
Не факт, что там формула есть. Я думал через динамику делать. 7 тестов верно. 8 - ошибка. Я так и не понял где косяк. ;(
Аноним 05/06/16 Вск 16:12:51  429655
>>429645
>>429626
Аноним 05/06/16 Вск 16:14:07  429663
>>429654
> Я думал через динамику делать.
Ебал моя рука. Мозг после этих 5 часов подохуел.
Аноним 05/06/16 Вск 16:15:27  429673
>>429649
сука я думал N =(
Аноним 05/06/16 Вск 16:16:58  429684
>>429631
отправил первую, вторую, четвертую, шестую целиком (но не осилил привести ответ в общем случае к красивому виду, хотя раньше дохуя таким занимался), 7.1 скорее наугад выбрал

не смог до ума довести третью, не успел пятую и седьмую, как-то много времени на хуйню всякую просрал

в общем, из 29 баллов набрал где-то 14-18, это, может и не полное SOSALITY, но и не успех

единственная надежда, что в мой мухосранский филиал маленький конкурс и такого убогого, как я, тоже возьмут

так-то похуй, в следующем году всяко получится
Аноним 05/06/16 Вск 16:21:48  429720
>>429684
какой город?
набрал гдето 13-15, тоже надеюсь на количество мест
Аноним 05/06/16 Вск 16:22:22  429723
>>429684
>из 29 баллов
Ты даже баллы посчитать не можешь.
Аноним 05/06/16 Вск 16:28:12  429771
>>429720
екб а у тебя?

>>429723
сорян, в шары ебусь, думал, что в последней 4, а в ней внезапно 5

Так что, аноны, максимальный балл - 30
Аноним 05/06/16 Вск 16:32:11  429798
>>429771
nsk
Аноним 05/06/16 Вск 16:32:45  429804
задачки такие простые, что даже неприлично их обсуждать в этом заумном треде
Аноним 05/06/16 Вск 16:34:01  429814
>>429771
Я не он, тот же город, результат 17-26. Закончивший в этом году друг говорил, что с половиной баллов на собеседование приглашают, но на нем проще если было больше баллов.
Аноним 05/06/16 Вск 16:44:50  429882
>>429684
в шад берут один раз. Если валишься один раз, следующую не пройдешь на собеседе, инфа 100%
Аноним 05/06/16 Вск 16:47:10  429900
>>429804
Все решил полностью?
Аноним 05/06/16 Вск 16:47:43  429905
>>429882
ху-е-та.
Аноним 05/06/16 Вск 17:41:22  430089
>>429882
Откуда инфа
Аноним 05/06/16 Вск 18:11:21  430209
>>430089
От закончивших ШАД. Любой, кто пытался поступить и не смог, попадает в спциальную табличку. Попавших в эту табличку валят на собеседованиях, так что в ШАД либо проходишь сразу, либо — никогда.
Аноним 05/06/16 Вск 18:20:02  430252
>>430209
ну не знаю. у нас в мухосранске точно не так. Знаю парня, который поступал два раза, со второго поступил. Какия им разница, поступал ты или нет?
Аноним 05/06/16 Вск 18:25:39  430282
1. pi/24
2. -2
3. 1/4
4. перебираем y, чтобы было до 10^6. потом проверяем, извлекается ли корень из n - yyу
5. объясните, кто чё, писал ебанутую дпшку, но валилась на последних тестах. хз, или набажил, или говнорешение
6.1 121/243
6.2 перебираем, как мы можем набрать чётную сумму. расписываем все варианты через сочетания, расставляя нечетные цифры между двойками. Разберем отдельно для чётных и нечётных, получим формулу из сумм цэшек. Если её закинуть в вольфрам, сворачивается в $ 1/2 * (1 + (-1 / 3) ^ n) $, если кто знает, как получить её сразу, отпишите.
7.1 7
7.2 ставим в середину log, в середины половинок log - 1 и т.д. получаем такую симметричную ёлочку. на ней изи доказать, что будет nlogn. но не успел доказать, что это худший случай.
Аноним 05/06/16 Вск 18:29:48  430301
есть кто из минска? много порешали?
в прошлом году, написано, что 24 человека взяли. это ж блин реально мало. все курсы фпми, мехмата бгу + половину бгуира, +магов, аспирантов. т.д.
как вообще попасть в эту 24-ку? чё на собесе будет?
Аноним 05/06/16 Вск 18:40:51  430369
>>430282
у меня точно такие же ответы :)
свернуть можно с помощью бинома Ньютона:
распиши (1+2)^n и (1-2)^n. Сложи два этих равенства, получишь двойную сумму из четных биномиальных коэффициентов. Это как раз и получиться (3^n-(-1)^n)/2
ну а потом раздели почленно на 3^n.
Аноним 05/06/16 Вск 18:45:46  430395
>>430282
> 2. -2

Можешь объяснить как? Я просто вводил каждое в вольфрам, сумма была равна -2, так и написал, но хотелось бы нормальное решение.

> 4. перебираем y, чтобы было до 10^6. потом проверяем, извлекается ли корень из n - yyу

По y можно было только до кубического корня из n идти, но если тесты проходит, то сойдет.

> 5. объясните, кто чё, писал ебанутую дпшку, но валилась на последних тестах. хз, или набажил, или говнорешение

Хуй знает, пытался разные комбинаторные формулы подбирать из того что похожесть - отношение эквивалентности, значит разбивает все множество строк на классы эквивалентности. Каждый класс = множество перестановок K элементов, значит в нем K! элементов. Всего строк длины L над алфавитом размера K будет K^L, отсюда надо ещё вычесть те строки, в которые входят не все K символов, это вроде K(K-1)^L. Общее число строк делим на размер класса и, кажется, получается то что нужно. Объясните ошибки, у меня валилось на 8 тесте.

> 6.2 перебираем, как мы можем набрать чётную сумму. расписываем все варианты через сочетания, расставляя нечетные цифры между двойками. Разберем отдельно для чётных и нечётных, получим формулу из сумм цэшек. Если её закинуть в вольфрам, сворачивается в $ 1/2
(1 + (-1 / 3) ^ n) $, если кто знает, как получить её сразу, отпишите.

По идее она как-то через кручения-верчения сочетаний и биномиального разложения и должна получаться. Мне и с суммой нормально.

> 7.2 ставим в середину log, в середины половинок log - 1 и т.д. получаем такую симметричную ёлочку. на ней изи доказать, что будет nlogn. но не успел доказать, что это худший случай.

Я только написал про этот пример, что в таком случае самый длинный из возможных внутренний цикл будет работать на полную длину и если рекурсивно рассмотреть половинки на них будет тоже самое. Нестрого, на какие-то баллы будут.
Аноним 05/06/16 Вск 18:45:51  430396
>>430369
получится (3^n+(-1)^n)/2 - плюс, не минус. ошиблась
Аноним 05/06/16 Вск 18:46:19  430399
14651415791600.jpg (976Кб, 3264x1836)
>>430282
> 3. 1/4
можешь написать какое множество у тебя получилось?
У меня пикрел, но я не уверен.
Аноним 05/06/16 Вск 18:47:51  430410
>>430399
у меня то же самое получилось
Аноним 05/06/16 Вск 18:49:44  430413
14651417844390.jpg (35Кб, 350x343)
>>430410
Бляяяяяяяяяяяяя. "Долю площади", а я площадь вписал.
Аноним 05/06/16 Вск 18:50:30  430417
>>430209
А в чем прикол-то такого подхода? И как можно на собеседовании завалить?
Аноним 05/06/16 Вск 19:00:06  430478
>>430209
Если так, то это конечно пиздец подстава. Так я бы и не пошел поступать в этом году.
Аноним 05/06/16 Вск 19:02:31  430489
>>430395
7.2 строго:
T(n) <= F_{1,n}(i) + T(i) + T(n-i) <= n + T(i) + T(n-i) <= n + 2 * T(n/2) = O(n log n)
Где i = это индекс максимального элемента в массиве a[1...n] ,
F_{1,n}(i) - это количество операций, которое ты проделаешь на i-ой итерации цикла FOR.

Ну а пример для достижения O(n log n) тут уже обсуждали
Аноним 05/06/16 Вск 19:03:52  430499
>>430478
явный же фейк. будете волноваться, сливать собес
Аноним 05/06/16 Вск 19:09:55  430524
>>430417
один кун завалился вопросом о мотивации(хотя может и пиздит, просто не решил таски, которые дают)
в этом году вообще хз, чё будет. мб проверят то, что не проверили на экзе. ну и вопросы на логику, надеюсь
Аноним 05/06/16 Вск 19:15:33  430549
>>430524
а сколько процентов с собеса обычно берут?
Аноним 05/06/16 Вск 19:38:51  430695
Повторю вопрос
Пацаны, кто решал вариант от 28го, посмотрите вторую задачу, про сходимость ряда. Там же ряд сходится, да? Или меня жестко глючит? Просто минус за эту задачу стоит.
Аноним 05/06/16 Вск 19:42:08  430705
>>430695
если ты про ту где куча логарифмов то да, сходится
Аноним 05/06/16 Вск 20:06:20  430790
>>430695
да, я ж тут решение даже написал.смотри выше в треде
Аноним 05/06/16 Вск 20:11:54  430813
>>430790
Да я тоже на экзамене решение на нее писал, и тоже написал, что сходится.
Короче то ли проверяющий напутал че-то, то ли я думал одно, а написал другое. Во втором случае канеш пиздец обидно будет.
Аноним 05/06/16 Вск 20:12:55  430817
>>430489
откуда последнее неравенство???
Аноним 05/06/16 Вск 20:14:11  430820
а че там про поступающих во второй раз правда? есть у кого знакомые, поступившие со второго и более раза?
Аноним 05/06/16 Вск 20:15:17  430822
>>430395
K(K-1)^L как получил?
Аноним 05/06/16 Вск 20:25:29  430861
>>430822
(K-1)^L - количество строк в которые входят только K-1 или меньше символов; умножение на K так как есть K вариантов того, какой символ исключаем из перебора.
Аноним 05/06/16 Вск 20:35:24  430895
>>430813
Еще блядь в третей задаче чисто по невнимательности накосячил и ответ неверный получил, перепутал плотности местами в интеграле. Если и во второй думал одно, а написал другое, то пиздец блядь будет, не поступить в ШАД даже не из-за арифметических ошибок, а из-за описок((
Аноним 05/06/16 Вск 20:43:24  430932
>>430895
бро, я знаю это чувство, сегодня писал онлайн-для-даунов, и тоже по невнимательности в одной задаче не то написал, хотя на бумаге решил правильно
Аноним 05/06/16 Вск 20:49:30  430959
А я вот решил правильно задачу, файл отправил. А вот пример неправильно вбил. И по правилам не проверят. Как думаете, можно ли выцепить баллы?
Аноним 05/06/16 Вск 21:44:12  431282
>>430861
ну это неверно, потому что твои множества пересекаются.
если ты выкинул один элемент, то в это множество входят также и последовательности с 2мя выброшенными элементами, 3мя и т.д. а эти последовательности входят и в другие множества
Аноним 05/06/16 Вск 22:40:45  431598
>>430817
Можно расписать подробнее.
T(n) = f_{1,n}(i) + T(i) + T(n-i) = 2min(i, n-i) + T(i) + T(n-i) = 2min(i, n-i) + (2min(i - i_1, i_1) + T(i) + T(i - i_1)) + (2min(i_2 - i, n - i_2) + T(i_2 - i) + T(n - i_2)) <= n + 2 (n/2) + 2(n/2) + T(i) + T(i - i_1) + T(i_2 - i) + T(n - i_2)
Где 1 <= i_1 < i < i_2 <= n (для n >= 3), i - индекс макс элемента на всем массиве, i_1 - индекс макс элемента на a[1..i-1], i_2 - индекс макс элемента на a[i+1, n].

Если раскрывать T(i_j) дальше, то станет понятно что сумма максимизируется при условии, что i = n-i, i - i_1 = i_1, i_2 - i = n - i_2 и т.д.,
то есть еслимаксимальные элементы стоят по середине соответсвующих им отрезков. В этом случае тут ассимптотика такая же как и у merge sort = O(n log n).
Аноним 06/06/16 Пнд 02:32:03  431930
А есть ещё треды про шад?
Хочу поговорить про 4 число
Аноним 06/06/16 Пнд 07:59:52  432263
>>431598
где таким можно обмазаться?
Аноним 06/06/16 Пнд 08:49:03  432326
>>431930
Поговори со мной
Аноним 06/06/16 Пнд 11:47:44  432371
>>430209
Я 2 уже не поступил. Сссука
Аноним 06/06/16 Пнд 11:49:10  432374
Надо было заочный варик писать.
Аноним 06/06/16 Пнд 15:05:27  432739
>>432326
Давай сверим ответы хотя бы
Аноним 06/06/16 Пнд 19:48:06  433755
14652316862280.png (41Кб, 925x448)
А как решать пикрилейтед? Чёта я решал-решал, не дорешал
Аноним 06/06/16 Пнд 19:56:52  433772
>>433755
Матрица подобна своей жордановой форме и жорданова форма единственна, значит условие можно перефразировать как существование вещественных собственных чисел. Пишешь характеристический многочлен, находишь условия на х, у.
Аноним 06/06/16 Пнд 20:20:35  433804
>>433755
Дополню этого >>433772
>как существование вещественных собственных чисел
как существование двух вещественных собственных чисел
Аноним 06/06/16 Пнд 20:53:52  433834
>>433772
Аноним 06/06/16 Пнд 21:26:58  433873
Аноны, как на собес проходят обычно? Три плюса и плюс минус или три плюса и минус плюс?
Аноним 07/06/16 Втр 03:16:17  434217
>>433873
какие еще плюсы-минусы?
Аноним 07/06/16 Втр 09:26:05  434287
>>434217
видимо имеется ввиду проходной бал
Аноним 07/06/16 Втр 11:01:32  434337
>>433873
по-моему считают только то, что с плюсом впереди. плюс-минус, плюс пополам. минус-плюс не считают
Аноним 07/06/16 Втр 12:18:21  434426
А как думаете, какой проходной будет в этом году?
Аноним 07/06/16 Втр 12:42:51  434449
>>434337
Херовато((
Аноним 07/06/16 Втр 12:43:40  434451
>>434426
С этого года вроде как на платное отделение тоже набирают. На экзамене 4-го июня было, кажется, немногим больше 200 человек. Надо сравнить с конкурсом прошлых лет.
Аноним 07/06/16 Втр 14:00:44  434640
В прошлом году было примерно столько же человек (судя по ощущениям). Проходной порог был 3 задачи. Но и задания были посложнее.
Хз как платное отделение скажется - уменьшить процент отбора на 3 этап или не изменит.
Аноним 08/06/16 Срд 20:55:05  437293
Сап
Может кто-то из студентов ШАДа поделиться домашними заданиями курса Воронцова по машинному обучению?
Аноним 08/06/16 Срд 21:39:49  437353
>>437293
кто такой Воронцов?
Аноним 08/06/16 Срд 22:02:34  437374
>>437353
Мы вам перезвоним.
Аноним 11/06/16 Суб 09:04:29  440621
Ща бы нихуя результаты не присылать.
Аноним 11/06/16 Суб 17:22:01  441074
А чё, уже можна паниковать начинать? Апелляция на носу, а результатов до сих пор нет :(
Аноним 11/06/16 Суб 18:07:39  441120
кто в минск поступает?
Аноним 12/06/16 Вск 11:04:25  441501
>>441120
я поступаю.
как ты будешь к собесу готовиться?

[Назад][Обновить тред][Вверх][Каталог] [Реквест разбана] [Подписаться на тред] [ ] 797 | 41 | 207
Назад Вверх Каталог Обновить

Топ тредов